Крок 1 - Медицина 2012 (буклет)

1 / 200
Метильні групи (—CH3) використовуються в органiзмi для синтезу таких важливих сполук, як креатин, холін, адреналін, інші. Джерелом цих груп є одна з незамінних амінокислот, а саме: Methyl groups (—CH3) are used in the body for the synthesis of such important compounds as creatine, choline, adrenaline, and others. The source of these groups is one of the essential amino acids, namely:

Метіонін Methionine

Ізолейцин Isoleucine

Триптофан Tryptophan

Лейцин Leucine

Валін Valin

2 / 200
Хвора 46-ти років скаржиться на сухість в роті, спрагу, почащений сечопуск, загальну слабкість. У крові: гіперглікемія, гіперкетонемія. У сечі: глюкоза, кетонові тіла. На ЕКГ: дифузні зміни в міокарді. Який найбільш імовірний діагноз? A 46-year-old patient complains of dry mouth, thirst, frequent urination, general weakness. Blood: hyperglycemia, hyperketonemia. Urine: glucose, ketone bodies . On the ECG: diffuse changes in the myocardium. What is the most likely diagnosis?

Гострий панкреатит Hostry Pancreatitis

Немічна хвороба серця Degenerative heart disease

Аліментарна гіперглікемія Alimentary hyperglycemia

Нецукровий діабет Diabetes insipidus

Цукровий діабет Diabetes

3 / 200
В шкірі виявлена щільна, рухома, чітко відмежована від оточуючих тканин пухлина. На розрізі вона білого кольору, представлена волокнистою тканиною. Мікроскопічно: хаотично переплетені колагенові волокна, клітин мало. Що це за пухлина? A dense, mobile tumor, clearly separated from the surrounding tissues, was detected in the skin. On cross-section, it is white in color, represented by fibrous tissue. Microscopically: chaotically intertwined collagen fibers, few cells . What kind of tumor is this?

Міома Myoma

Десмоїд Desmoid

Дерматофіброма Dermatofibroma

Фіброма Fibroma

Гістіоцитома Histiocytoma

4 / 200
У постраждалого виявлено рану верхньої частини передньої ділянки плеча. При обстеженні встановлена втрата активного згинання в ліктьовому суглобі і чутливості шкіри передньобічної поверхні передпліччя. Порушення функції якого нерва має місце? The victim was found to have a wound in the upper part of the front part of the shoulder. During the examination, the loss of active flexion in the elbow joint and the sensitivity of the skin of the anterolateral surface of the forearm was detected. What nerve function is impaired?'

М’язово-шкірний Musculocutaneous

Пахвовий Axillary

Променевий Radial

Серединний Middle

Ліктьовий Cubit

5 / 200
При підозрі на туберкульоз хворій дитині зробили пробу Манту. Через 24 години у місці введення алергену з’явились припухлість, гіперемія і болісність. Які основні компоненти визначають цю реакцію організму? On suspicion of tuberculosis, a sick child was given a Mantoux test. After 24 hours, swelling, hyperemia, and pain appeared at the site of allergen injection. What are the main components that determine this reaction of the body ?

В-лімфоцити, IgМ B-lymphocytes, IgM

Плазматичні клітини, Т-лімфоцити і лімфокіни Plasma cells, T-lymphocytes and lymphokines

Макрофаги, В-лімфоцити і моноцити Macrophages, B-lymphocytes and monocytes

Гранулоцити, Т-лімфоцити і IgG Granulocytes, T-lymphocytes and IgG

Мононуклеари, Т-лімфоцити і лімфокіни Mononuclear cells, T-lymphocytes and lymphokines

6 / 200
На прийом до терапевта прийшов чоловік 37-ми років зі скаргами на періодичні інтенсивні больові напади у суглобах великого пальця стопи та їх припухлість. У сечі: різко кисла реакція і рожеве забарвлення. З наявністю яких речовин можуть бути пов’язані такі зміни? A 37-year-old man came to see a therapist with complaints of periodic intense pain attacks in the joints of the big toe and their swelling. In the urine: a sharp acidic reaction and pink color. With the presence of what substances can such changes be associated?

Хлориди Chlorides

Амонієві солі Ammonium salts

Солі сечової кислоти Uric acid salts

Фосфат кальцію Calcium phosphate

Сульфат магнію Magnesium Sulfate

7 / 200
При санітарно-бактеріологічному дослідженні водопровідної води отримані наступні результати: загальна кількість бактерій в 1,0 мл - 80, колі-індекс - 3. Як розцінити результат дослідження? During the sanitary-bacteriological examination of tap water, the following results were obtained: the total number of bacteria in 1.0 ml is 80, the coli index is 3. How to evaluate the result of the study?

Вода є забрудненою The water is polluted

Вода є дуже сумнівною The water is very questionable

Вода є дуже забрудненою The water is very polluted

Вода є сумнівною Water is questionable

Вода придатна для споживання Water is suitable for consumption

8 / 200
в крові: ер.- 3,0 • 1012 8. /л; Hb-90г/л; ретикул.- 0,5%. В мазку: пойкілоцити, гіпохромні еритроцити. Залізо сироватки крові - 80 мкмоль/л. Для якої патології це характерно? in the blood: er.- 3.0 • 1012 8. /l; Hb-90g/l; reticul.- 0.5%. In the smear: poikilocytes, hypochromic erythrocytes. Blood serum iron - 80 μmol/l. What pathology is this typical for?

Хвороба Мінковського-Шоффара Minkowski-Shofar disease

Залізодефіцитна анемія Iron deficiency anemia

Серпоподібноклітинна анемія Sickle cell anemia

B12-дефіцитна анемія B12-deficiency anemia

Залізорефрактерна анемія Iron refractory anemia

9 / 200
Хворий 40-ка років впродовж тривалого часу страждає на бронхіальну астму і тахікардію. Вкажіть найдоцільніший в даній ситуації препарат для усунення бронхоспазму: A 40-year-old patient suffers from bronchial asthma and tachycardia for a long time. Indicate the most appropriate drug to eliminate bronchospasm in this situation:

Атропіну сульфат Atropine sulfate

Ефедрину гідрохлорид Ephedrine hydrochloride

Адреналіну гідрохлорид Adrenaline hydrochloride

Сальбутамол Salbutamol

Ізадрин Izadrin

10 / 200
Хворому на гіпертонічну хворобу був призначений препарат для зниження артеріального тиску з групи адренотропних засобів. Через деякий час у хворого тиск нормалізувався, але розвинулися брадикардія до 50/хв. та атріовентрикулярна блокада II ступеня. Який препарат було призначено? A hypertensive patient was prescribed a drug to lower blood pressure from the group of adrenotropic agents. After some time, the patient's blood pressure normalized, but bradycardia up to 50/min and atrioventricular blockade of the II degree. What drug was prescribed?

Верапаміл Verapamil

Мезатон Мезатон

Анаприлін Анаприлін

Празозин Prazosin

Клофелін Clofelin

11 / 200
Хлопчик 12-ти років знаходиться у лікарні з підозрою на харчову токсикоінфекцію. При посіві фекалій хворого на середовище Ендо виросла велика кількість безбарвних колоній. Який мікроорганізм можна з найбільшою імовірністю ВИКЛЮЧИТИ з числа можливих збудників захворювання? A 12-year-old boy is in the hospital with suspected food poisoning. A large number of colorless colonies grew when the patient's feces were cultured on Endo's medium. Which microorganism is most likely EXCLUDE from the number of possible causative agents of the disease?

Proteus vulgaris Proteus vulgaris

Escherichia coli Escherichia coli

Pseudomonas aeruginosa Pseudomonas aeruginosa

Yersinia enterocolitica Yersinia enterocolitica

Salmonella enteritidis Salmonella enteritidis

12 / 200
Хворій 65-ти років, що страждає на інсулінонезалежний цукровий діабет, призначили всередину глібенкламід. Вкажіть механізм гіпоглікемічної дії цього препарату: A 65-year-old patient suffering from non-insulin-dependent diabetes was prescribed glibenclamide orally. State the mechanism of hypoglycemic action of this drug:

Посилює утилізацію глюкози периферичними тканинами Strengthens utilization of glucose by peripheral tissues

Пригнічує глюконеогенез у печінці Suppresses gluconeogenesis in the liver

Стимулює виділення ендогенного інсуліну бета-клітинами Stimulates release of endogenous insulin by beta cells

Пригнічує всмоктування глюкози у кишечнику Suppresses the absorption of glucose in the intestine

Пригнічує альфа-глюкозидазу і розпад полісахаридів Suppresses alpha-glucosidase and breakdown of polysaccharides

13 / 200
У тварини через 2 тижні після експериментального звуження ниркової артерії підвищився артеріальний тиск. Зі збільшенням дії на судини якого фактора гуморальної регуляції це пов’язано? 2 weeks after the experimental narrowing of the renal artery, the animal's blood pressure increased. This is due to an increase in the action of which factor of humoral regulation on vessels?

Альдостерон Aldosterone

Вазопресин Vasopressin

Ангіотензин II Angiotensin II

Дофамін Dopamine

Кортизол Cortisol

14 / 200
У лабораторному експерименті на собаці вивчали будову центральних відділів слухової сенсорної системи. Була зруйнована одна з структур середнього мозку. Собака втратив орієнтувальний рефлекс на звукові сигнали. Яка структура була зруйнована? In a laboratory experiment on a dog, the structure of the central parts of the auditory sensory system was studied. One of the structures of the midbrain was destroyed. The dog lost its orientation reflex to sound signals. Which structure was destroyed ?

Верхні горбики чотиригорбикового тіла Upper tubercles of quadrituberous body

Червоне ядро Red core

Чорна речовина Black Matter

Нижні горбики чотиригорбикового тіла Inferior tubercles of quadrituberous body

Ядра ретикулярної формації Nuclei of reticular formation

15 / 200
Хворий 23-х років надійшов до лікарні із черепно-мозковою травмою у важкому стані. Дихання характеризується судомним тривалим вдихом, який переривається коротким видихом. Для якого типу дихання це характерно? A 23-year-old patient was admitted to the hospital with a brain injury in a serious condition. Breathing is characterized by a convulsive long inhalation, which is interrupted by a short exhalation. For what type of breathing is this typical?

Біота Biota

Чейн-Стокса Cheyne-Stokes

Гаспінг-дихання Gasping Breath

Куссмауля Kussmaul

Апнейстичне Apneistic

16 / 200
При токсичному ушкодженні клітин печінки з порушенням її функцій у хворого з’явилися набряки. Які зміни складу плазми крові є провідною причиною розвитку набряків? Due to toxic damage to liver cells with impaired liver function, the patient developed edema. What changes in blood plasma composition are the leading cause of edema development?

Зменшення вмісту глобулінів Decreasing globulin content

Зменшення вмісту фібриногену Decrease in fibrinogen

Збільшення вмісту альбумінів Increase in albumin content

Зниження вмісту альбумінів Decreasing albumin content

Збільшення вмісту глобулінів Increase in globulin content

17 / 200
Під час операції в печінці хворого виявлені дрібні міхурці малих розмірів з незначною кількістю рідини, які щільно прилягають один до одного. Який гельмінтоз виявився у хворого? During the operation in the patient's liver, small vesicles of small sizes with a small amount of liquid were found, which tightly adhere to each other. What kind of helminthiasis was found in the patient?

Дікроцеліоз Dicroceliosis

Клонорхоз Clonorchiasis

Фасціольоз Fasciolosis

Опісторхоз Opistorchosis

Альвеококоз Alveococcus

18 / 200
Тварині, сенсибілізованій туберкуліном, внутрішньоочеревенно введений туберкулін. Через 24 години при лапаротомії виявлено венозну гіперемію та набряк очеревини. У мазках-відбитках з очеревини велика кількість лімфоцитів та моноцитів. Який патологічний процес у тварини? In an animal sensitized by tuberculin, tuberculin was injected intraperitoneally. After 24 hours, at laparotomy, venous hyperemia and edema of the peritoneum were found. In smears-prints from the peritoneum, a large number of lymphocytes and monocytes. What is the pathological process in the animal?

Фібринозне запалення Fibrinous inflammation

Алергічне запалення Allergic inflammation

Серозне запалення Serous inflammation

Асептичне запалення Aseptic inflammation

Гнійне запалення Purulent inflammation

19 / 200
У хлопчика 3-х років з вираженим геморагічним синдромом відсутній антигемофільний глобулін А (фактор VIII) у плазмі крові. Яка фаза гемостазу первинно порушена у цього хворого? A 3-year-old boy with severe hemorrhagic syndrome has no antihemophilic globulin A (factor VIII) in his blood plasma. Which phase of hemostasis is primarily disturbed in this patient?

Внутрішній механізм активації протромбінази Internal mechanism of prothrombinase activation

Перетворення фібриногену в фібрин Conversion of fibrinogen to fibrin

Зовнішній механізм активації протромбінази External mechanism of prothrombinase activation

Ретракція кров’яного згустку Blood clot retraction

Перетворення протромбіну в тромбін Conversion of prothrombin to thrombin

20 / 200
У пацієнта з бронхіальною астмою за допомогою шкірних алергічних проб встановлено сенсибілізацію алергеном тополиного пуху. Який фактор імунної системи відіграє вирішальну роль в розвитку цього імунопатологічного стану? A patient with bronchial asthma was diagnosed with poplar down allergen sensitization using skin allergy tests. What factor of the immune system plays a decisive role in the development of this immunopathological condition?

IgG IgG

Сенсибілізовані Т-лімфоцити Sensitized T-lymphocytes

IgD IgD

IgE IgE

IgM IgM

21 / 200
При недостатності кровообігу у період інтенсивної м’язової роботи у м’язі в результаті анаеробного гліколізу накопичується молочна кислота. Яка її подальша доля? With insufficient blood circulation during the period of intense muscle work, lactic acid accumulates in the muscle as a result of anaerobic glycolysis. What is its further fate?

Видаляється через нирки з сечею Excreted through the kidneys with urine

Використовується у тканинах для синтезу жирних кислот Used in tissues for the synthesis of fatty acids

Включається в глюконеогенез у печінці Involved in gluconeogenesis in the liver

Використовується у м’язі для синтезу амінокислот Used in muscle to synthesize amino acids

Використовується тканинами для синтезу кетонових тіл Used by tissues to synthesize ketone bodies

22 / 200
Дитина 10-ти років страждає на стафілококовий дерматит. Лікування бензилпеніциліном не дало результатів. Призначення комбінованого препарату пеніциліну з клавулановою кислотою дало швидке одужання. Яка причина позитивної дії цього препарату? A 10-year-old child suffers from staphylococcal dermatitis. Treatment with benzylpenicillin did not give results. The appointment of a combined penicillin with clavulanic acid resulted in a quick recovery. What is the reason for the positive effect of this drug ?

Блокада транслокази Translocasi Blockade

Гальмування аденозиндезамінази Inhibition of adenosine deaminase

Гальмування транспептидази Inhibition of transpeptidase

Активація фосфодіестерази Activation of phosphodiesterase

Інактивація бета-лактамази Beta-lactamase inactivation

23 / 200
Хворий 18-ти років звернувся до лікаря зі скаргами на безсоння, що проявляється у важкому засинанні. В результаті цього він не висипається і на наступний день почуває втому, важко засвоює навчальний матеріал. Лікар встановив, що безсоння пов’язане з неврозоподібним станом. Зробіть раціональний вибір снодійного: An 18-year-old patient turned to the doctor with complaints of insomnia, manifested in difficulty falling asleep. As a result, he does not get enough sleep and the next day he feels tired, difficult learns educational material. The doctor established that insomnia is related to a neurosis-like condition. Make a rational choice of a sleeping pill:

Хлоралгідрат Chloral hydrate

Бромізовал Bromisoval

Нітразепам Nitrazepam

Етамінал-натрій Ethaminal sodium

Фенобарбітал Phenobarbital

24 / 200
У хворого, який страждає на вугрі та на запальні зміни шкіри обличчя, при мікроскопії матеріалу з осередків ураження виявлені живі членистоногі, довгастої форми, з 4 парами дуже редукованих кінцівок. Встановіть попередній діагноз: In a patient who suffers from acne and inflammatory changes of the skin of the face, microscopy of the material from the lesions revealed living arthropods, elongated in shape, with 4 pairs of very reduced limbs Establish a preliminary diagnosis:

Ураження шкіри коростяним свербуном Skin lesions with scabies

Алергія Allergy

Ураження шкіри блохами Skin infestation with fleas

Педикульоз Pediculosis

Демодекоз Demodecosis

25 / 200
Подразнення правого блукаючого нерва спричинило різке сповільнення атріовентрикулярного проведення. На ЕКГ при цьому буде подовжений: Irritation of the right vagus nerve caused a sharp slowing of the atrioventricular conduction. In this case, the ECG will be prolonged:

Зубець T Teeth T

Інтервал P — Q Interval P — Q

Комплекс QRST QRST Complex

Зубець P P tooth

Інтервал R — R Interval R — R

26 / 200
У людини після укусу москітом виникли виразки шкіри. Аналіз вмісту виразки виявив всередині клітин людини безджгутикові одноклітинні організми. Який попередній діагноз? A person developed skin ulcers after a mosquito bite. Analysis of the contents of the ulcer revealed non-flagellated unicellular organisms inside the human cells. What is the preliminary diagnosis?

Лейшманіоз вісцеральний Visceral leishmaniasis

Балантидіоз Balantidiosis

Токсоплазмоз Toxoplasmosis

Лейшманіоз дерматотропний Leishmaniasis dermatotropic

Трипаносомоз Trypanosomosis

27 / 200
У чоловіка 63-х років рак стравоходу, метастази у лімфатичні вузли середостіння, ракова кахексія. Яка патогенетична стадія пухлинного процесу має місце? A 63-year-old man has esophageal cancer, metastases in the mediastinal lymph nodes, cancer cachexia. What is the pathogenetic stage of the tumor process?

Промоції Promotions

Трансформації Transformations

Ініціації Initiations

Прогресії Progressions

- -

28 / 200
Жінку 44-х років вжалила оса, внаслідок чого розвинувся шок. В анамнезі - тяжка алергічна реакція на жалення оси. Об’єктивно: Ps- 179/хв, слабкий, АТ- 80/40 мм рт.ст., ЧД- 26/хв. Яка провідна ланка патогенезу анафілактичного шоку? A 44-year-old woman was stung by a wasp, as a result of which she developed shock. She has a history of a severe allergic reaction to a wasp sting. Objectively: Ps- 179/min, weak, blood pressure - 80/40 mm Hg, blood pressure - 26/min. What is the leading link in the pathogenesis of anaphylactic shock?

Зменшення ударного об’єму серця Decreased stroke volume of the heart

Тахікардія Tachycardia

Зменшення об’єму циркулюючої крові Decreasing the volume of circulating blood

Зниження периферійного опору судин Reduction of peripheral vascular resistance

Біль Pain

29 / 200
Хворий надійшов до інфекційного відділення з підозрою на холеру. Який основний метод дослідження необхідно використати для підтвердження діагнозу? The patient was admitted to the infectious disease department with suspicion of cholera. What basic research method should be used to confirm the diagnosis?

Серологічний Serological

Бактеріологічний Bacteriological

Біологічний Biological

Імунологічний Immunological

Алергічний Allergic

30 / 200
При розтині тіла померлого виявлена гіперплазія кісткового мозку плоских і трубчастих кісток (піоїдний кістковий мозок), спленомегалія (6 кг), гепатомегалія (5 кг), збільшення всіх груп лімфатичних вузлів. Якому захворюванню відповідають виявлені зміни? During the autopsy of the deceased, hyperplasia of the bone marrow of flat and tubular bones (pyoid bone marrow), splenomegaly (6 kg), hepatomegaly (5 kg), an increase in all groups was found lymph nodes. What disease do the detected changes correspond to?

Справжня поліцитемія Polycythemia True

Лімфогрануломатоз Lymphogranulomatosis

Хронічний мієлолейкоз Chronic myelogenous leukemia

Хронічний лімфолейкоз Chronic lymphocytic leukemia

Мієломна хвороба Myeloma

31 / 200
Електрофоретичне дослідження сироватки крові хворого на пневмонію показало збільшення однієї з білкових фракцій. Вкажіть її: Electrophoretic examination of the blood serum of a patient with pneumonia showed an increase in one of the protein fractions. Specify it:

β-глобуліни β-globulins

γ-глобуліни γ-globulins

α1-глобуліни α1-globulins

α2-глобуліни α2-globulins

Альбуміни Albumini

32 / 200
У хворого коса пахвинна грижа. Яке анатомічне утворення стало слабким місцем передньої черевної стінки? The patient has an oblique inguinal hernia. What anatomical formation became the weak point of the anterior abdominal wall?

Медіальна пахвинна ямка Medial inguinal fossa

Стегнова ямка Fotomas

Латеральна пахвинна ямка Lateral inguinal fossa

Надміхурова ямка Nadmikhur's Fossa

Пахвинний трикутник Inguinal triangle

33 / 200
У хворого запалення легень ускладнилось ексудативним плевритом. В якому з перелічених анатомічних утворень переважно може накопичуватися рідина? The patient's lung inflammation was complicated by exudative pleurisy. In which of the listed anatomical formations is the fluid most likely to accumulate?

Sinus costomediastinalis pleurae Costamediastinal pleural sinus

Sinus phrenicomediastinalis pleurae Sinus phrenicomediastinalis pleurae

Sinus transversus pericardii Transverse pericardial sinus

Sinus obliquus pericardii Oblique sinus of the pericardium

Sinus costodiaphragmaticus pleurae Sinus costodiaphragmaticus pleurae

34 / 200
Хворий не може відвести від тулуба верхню кінцівку. Який м’яз не виконує свою функцію? The patient cannot move the upper limb away from the trunk. Which muscle does not perform its function?

Малий круглий м’яз Small round muscle

Найширший м’яз спини Latisus dorsi

Дельтоподібний Delta

Підосний м’яз Plates muscle

Великий круглий м’яз Large round muscle

35 / 200
У хворої 45-ти років при електрокардіографічному обстеженні виявлено такі зміни: інтервал P — Q подовжений, при цьому випадає кожен другий або третій комплекс QRST. Яке саме порушення провідності серця спостерігається? In a 45-year-old patient, electrocardiographic examination revealed the following changes: the P — Q interval is prolonged, while every second or third QRST complex is lost. What exactly is the conduction disorder the heart is observed?

Атріовентрикулярна блокада I ступеня Atrioventricular block I degree

Синоаурікулярна блокада Sino-auricular blockade

Внутрішлуночкова блокада Intraventricular block

Атріовентрикулярна блокада повна Atrioventricular blockade is complete

Атріовентрикулярна блокада III ступеня Atrioventricular block III degree

36 / 200
У хворого порушена моторна функція язика. З патологією якого нерва це пов’язано? The patient has impaired motor function of the tongue. What nerve pathology is this associated with?

Блукаючий Wandering

Язикоглотковий Lingopharyngeal

Лицевий Licevius

Під’язиковий Sublingual

Додатковий Additional

37 / 200
При розтині тіла померлого чоловіка 65-ти років, який страждав на захворювання легень, патологічний процес переважно був локалізований у бронхах, де при гістологічному дослідженні були чітко видні залози, хрящові острівці та багаторядний циліндричний миготливий епітелій. В яких бронхах відбулися зміни? At the autopsy of the body of a deceased 65-year-old man who suffered from lung disease, the pathological process was mainly localized in the bronchi, where glands were clearly visible during histological examination, cartilage islands and multi-rowed cylindrical ciliated epithelium. In which bronchi have changes occurred?

Великі бронхи Large bronchi

Головні бронхи Main bronchi

Малі бронхи Small bronchi

Середні бронхи Medium bronchi

Термінальні бронхіоли Terminal bronchioles

38 / 200
До лікарні надійшов 9-річний хлопчик розумово і фізично відсталий. При біохімічному дослідженні крові: підвищена кількість фенілаланіну. Блокування якого ферменту може призве- сти до такого стану? A 9-year-old mentally and physically retarded boy was admitted to the hospital. During a biochemical blood test: an increased amount of phenylalanine. Blocking which enzyme can lead to this condition?

Оксидаза гомогентизинової кислоти Homogentisic acid oxidase

Фенілаланін-4-монооксигеназа Phenylalanine-4-monooxygenase

Аспартатамінотрансфераза Aspartate aminotransferase

Глутаматдекарбоксилаза Glutamate decarboxylase

Глутамінтрансаміназа Glutamine transaminase

39 / 200
За клінічними показами хворому призначено піридоксальфосфат. Для корекції яких процесів рекомендований цей препарат? According to clinical indications, pyridoxal phosphate is prescribed to the patient. For the correction of which processes is this drug recommended?

Дезамінування амінокислот Amino acid deamination

Синтез пуринових і піримідинових основ Synthesis of purine and pyrimidine bases

Окисне декарбоксилювання кетокислот Oxidative decarboxylation of ketoacids

Синтез білка Protein synthesis

Трансамінування і декарбоксилюван-ня амінокислот Transamination and decarboxylation of amino acids

40 / 200
До реанімаційного відділення надійшов хворий з ознаками гострого отруєння морфіном. Який засіб необхідно використати у даному випадку для промивання шлунку? A patient was admitted to the intensive care unit with signs of acute morphine poisoning. What agent should be used in this case for gastric lavage?

Розчин натрію хлориду Sodium chloride solution

Фурацилін Furacilin

Калію перманганат Potassium permanganate

Борна кислота Bornic acid

Натрію гідрокарбонат Sodium bicarbonate

41 / 200
У хворого відзначаються болі у ділянці кореня язика, зіву, піднебінних мигдаликів, у верхньому відділі глотки, вусі, втрачений смак у ділянці задньої третини язика. Ураженням якого нерва викликані ці порушення? The patient has pain in the area of the root of the tongue, pharynx, palatine tonsils, in the upper part of the pharynx, ears, lost taste in the area of the back third of the tongue. Damage to which nerve is caused by these violations?

Великий кам’янистий Big Rocky

Барабанна струна Drum string

Язиковий Language

Язикоглотковий Lingopharyngeal

Блукаючий Wandering

42 / 200
Для лікування деяких інфекційних захворювань, викликаних бактеріями, застосовуються сульфаніламідні препарати, що блокують синтез фактора росту бактерій. Назвіть механізм їх дії: For the treatment of some infectious diseases caused by bacteria, sulfonamide drugs are used that block the synthesis of the bacterial growth factor. Name the mechanism of their action:

Є алостеричними інгібіторами ферментів Are allosteric enzyme inhibitors

Є антивітамінами параамінобензойної кислоти Are para-aminobenzoic acid antivitamins

Є алостеричними ферментами Are allosteric enzymes

Беруть участь в окисно-відновних процесах Participate in redox processes

Інгібують всмоктування фолієвої кислоти Inhibit the absorption of folic acid

43 / 200
В сечі новонародженого визначається цитрулін та високий рівень амоніаку. Вкажіть, утворення якої речовини, найімовірніше, порушене у цього малюка: Citrulline and a high level of ammonia are detected in the urine of a newborn. Indicate the formation of which substance is most likely disturbed in this baby:

Сечова кислота Uric acid

Амоніак Ammonia

Креатинін Creatinine

Креатин Creatine

Сечовина Urea

44 / 200
Жінка 45 -ти років, перукар, скаржиться на болі у ногах, що з’являються після роботи, ввечері та вночі. При огляді хворої виявлено варикозне розширені вени на присередній поверхні гомілки та стегна. Яка вена та її безпосередні протоки розширені? A 45-year-old woman, a hairdresser, complains of pain in her legs that appear after work, in the evening and at night. During the patient's examination, varicose veins on on the medial surface of the lower leg and thigh. Which vein and its immediate ducts are dilated?

Глибока вена стегна Deep thigh vein

Стегнова Thigh

Мала підшкірна Small subcutaneous

Велика підшкірна Large subcutaneous

Передня великогомілкова вена Anterior tibial vein

45 / 200
У людини зменшений діурез, гіпернатріємія, гіпокаліємія. Гіперсекреція якого гормону може бути причиною таких змін? A person has reduced diuresis, hypernatremia, hypokalemia. Hypersecretion of which hormone can cause such changes?

Передсердний натрійуретичний фактор Atrial natriuretic factor

Адреналін Adrenaline

Вазопресин Vasopressin

Паратгормон Parathormone

Альдостерон Aldosterone

46 / 200
Після травми передньої поверхні верхньої третини передпліччя у хворого утруднення пронації, послаблення долонного згинання кисті та порушення чутливості шкіри 1-3 пальців на долоні. Який нерв ушкоджено? After an injury to the front surface of the upper third of the forearm, the patient has difficulty in pronation, weakening of the palmar flexion of the hand and impaired skin sensitivity of 1-3 fingers on the palm. What nerve is damaged?

n. musculocutaneus n. musculocutaneus

n. cutaneus antebrachii medialis medial forearm skin no.

n. radialis n. radialis

n. medianus n. medianus

n. ulnaris n. ulnaris

47 / 200
У хворого 45-ти років при аналізі ЕКГ встановлено: ритм синусовий, число передсердних комплексів більше числа шлуночкових комплексів; прогресуюче подовження інтервалу P — Q від комплексу до комплексу; випадіння окремих шлуночкових комплексів; зубці P та комплекси QRST без змін. Назвіть тип порушення серцевого ритму: In a 45-year-old patient, ECG analysis revealed: sinus rhythm, the number of atrial complexes is greater than the number of ventricular complexes; progressive lengthening of the P — Q interval from complex to complex; failure of individual ventricular complexes; P waves and QRST complexes are unchanged. Name the type of heart rhythm disorder:

Синоаурікулярна блокада Sino-auricular blockade

Атріовентрикулярна блокада I ступеня Atrioventricular block I degree

Атріовентрикулярна блокада II ступеня Atrioventricular block II degree

Внутрішньопередсердна блокада Intra-atrial blockade

Повна атріовентрикулярна блокада Complete atrioventricular block

48 / 200
В родині зростає дочка 14-ти років, у якої спостерігаються деякі відхилення від норми: зріст нижче, ніж у однолітків, відсутні ознаки статевого дозрівання, шия дуже коротка, плечі широкі. Інтелект в нормі. Яке захворювання можна припустити? The family has a 14-year-old daughter who has some deviations from the norm: her height is lower than that of her peers, there are no signs of puberty, her neck is very short, shoulders are broad. Intelligence is normal. What disease can be assumed?

Синдром Шерешевського-Тернера Shereshevsky-Turner syndrome

Синдром Едвардса Edwards Syndrome

Синдром Патау Patau Syndrome

Синдром Клайнфельтера Klinefelter Syndrome

Синдром Дауна Down Syndrome

49 / 200
Після обстеження хворому на сечокам’яну хворобу призначили алопурінол - конкурентний інгібітор ксантиноксидази. Підставою для цього був хімічний аналіз ниркових каменів, переважною складовою яких є: After an examination, a patient with urolithiasis was prescribed allopurinol - a competitive inhibitor of xanthine oxidase. The basis for this was a chemical analysis of kidney stones, the predominant component of which is:

Моногідрат оксалату кальцію Calcium oxalate monohydrate

Сульфат кальцію Calcium sulfate

Фосфат кальцію Calcium phosphate

Дигідрат оксалату кальцію Calcium oxalate dihydrate

Урат натрію Sodium urate

50 / 200
У хворого 60-ти років, що багато років страждає на атеросклероз і переніс раніше інфаркт міокарда, розвинувся напад загрудинного болю. Хворий госпіталізований через 3 дні; на фоні прогресуючої серцево-судинної недостатності помер. Під час розтину тіла у ділянці задньої стінки лівого шлуночка і міжшлуночкової перегородки виявлена ділянка білого кольору близько 3 см у діаметрі, волокниста, западаюча, з чіткою межею. Прозектор трактував ці зміни як: A 60-year-old patient who has suffered from atherosclerosis for many years and previously suffered a myocardial infarction developed an attack of chest pain. The patient was hospitalized 3 days later; on the background of progressive died of cardiovascular failure. During the autopsy, a white area about 3 cm in diameter, fibrous, sunken, with a clear border was found in the area of the back wall of the left ventricle and interventricular septum. The examiner interpreted these changes as:

Ішемія міокарда Myocardial ischemia

Дистрофія міокарда Myocardial dystrophy

Вогнищевий кардіосклероз Focal cardiosclerosis

Міокардит Myocarditis

Інфаркт міокарда Myocardial infarction

51 / 200
Дослідник при мікроскопічному і електронно-мікроскопічному вивченні печінки звернув увагу, що деякі окремо розташовані клітини розпалися на дрібні фрагменти, оточені мембраною. У деяких з них наявні органели, інші включають фрагменти ядра, що розпалося. Запальна реакція навколо відсутня. Дослідник розцінив ці зміни, як: During the microscopic and electron-microscopic study of the liver, the researcher noticed that some separately located cells broke up into small fragments surrounded by a membrane. Some of them have organelles, others include fragments of a disintegrated nucleus. There is no surrounding inflammatory response. The researcher interpreted these changes as:

Некроз Necrosis

Дистрофія Dystrophy

Атрофія Atrophy

Апоптоз Apoptosis

Гіпоплазія Hypoplasia

52 / 200
У хворого на підгострий септичний ендокардит при огляді лікар відзначив загальну слабкість і іктеричність шкіри, склер і видимих слизових оболонок. У крові виявлена збільшена кількість непрямого білірубіну. Що зумовлює жовтяничність шкіри і слизових? In a patient with subacute septic endocarditis, during the examination, the doctor noted general weakness and jaundice of the skin, sclera, and visible mucous membranes. An increased amount of indirect bilirubin was detected in the blood. What causes jaundice skin and mucous membranes?

Підпечінкова жовтяниця Subhepatic jaundice

Жирова дистрофія Fat dystrophy

Гемосидероз Hemosiderosis

Печінкова жовтяниця Hepatic Jaundice

Надпечінкова жовтяниця Suprahepatic jaundice

53 / 200
У хворого в анамнезі: з дитинства відмічався знижений рівень гемоглобіну. Лікування препаратами заліза не дає ефекту. У крові: ер.- 3,1 • 1012/л, ретик.-16%, Hb- 85 г/л, КП0,75; в мазку крові анізоцити, пойкілоцити, мішенеподібні еритроцити, еритроцити з базофільною зернистістю, рівень заліза у сироватці 30 мкмоль/л. Для якої патології системи крові характерні такі дані? The patient has a history of: a low hemoglobin level has been noted since childhood. Treatment with iron preparations has no effect. In the blood: er.- 3.1 • 1012/l, retic.-16%, Hb- 85 g/l, KP0.75, anisocytes, poikilocytes, target-like erythrocytes, erythrocytes with basophilic granules, iron level in serum 30 μmol/l in the blood smear. For which pathology of the blood system are these data characteristic?

Гіпопластична анемія Hypoplastic anemia

Залізодефіцитна анемія Iron deficiency anemia

Фолієводефіцитна анемія Folate deficiency anemia

B12-дефіцитна анемія B12-deficiency anemia

Таласемія Thalassemia

54 / 200
До приймального відділення лікарні доставлено хворого з вираженими явищами гострої серцевої недостатності. Який препарат, із зазначених нижче, слід використати в першу чергу? A patient with pronounced symptoms of acute heart failure was brought to the hospital's reception department. Which drug from the following should be used first?

Етазол Етазол

Фізостигмін Physostigmine

Дитилін Дитилін

Дигітоксин Digitoxin

Корглікон Корглікон

55 / 200
Чоловік 30-ти років звернувся до стоматолога зі скаргою на розлади жування, у нього виникає біль при відтягуванні щелепи назад. Запалення якого з жувальних м’язів найімовірніше встановить лікар? A 30-year-old man went to the dentist with a complaint of chewing disorders, he has pain when pulling his jaw back. The doctor will most likely diagnose the inflammation of the masticatory muscles ?

M. pterygoideus medialis M. pterygoideus medialis

M. pterygoideus lateralis M. pterygoideus lateralis

M. temporalis (задні волокна) M. temporalis (posterior fibers)

M. Мasseter M. Мasseter

M. temporalis (передш волокна) M. temporalis (front fibers)

56 / 200
Під час оперативного втручання на фоні використання гігронію різко знизився артеріальний тиск. Представники яких груп лікарських препаратів можуть нормалізувати артеріальний тиск? During surgical intervention against the background of using hygronium, blood pressure dropped sharply. Representatives of which drug groups can normalize blood pressure?

α-адреноміметики α-adrenomimetics

M-холіноміметики M-cholinomimetics

Гангліоблокатори Ganglioblockers

Н-холіноміметики H-cholinomimetics

α-адреноблокатори α-blockers

57 / 200
з дихальною недостатністю рН крові 7,35. Визначення pCO2 57. показало наявність гіперкапнії. При дослідженні рН сечі відзначається підвищення її кислотності. Яка форма порушення кислотно-основного стану в даному випадку? with respiratory insufficiency, blood pH 7.35. The determination of pCO2 57. showed the presence of hypercapnia. When examining the pH of urine, an increase in its acidity is noted. What is the form of violation of the acid-base state in this case?

Алкалоз газовий, компенсований Gas alkalosis, compensated

Ацидоз газовий, компенсований Gas acidosis, compensated

Алкалоз газовий, декомпенсований Gas alkalosis, decompensated

Ацидоз метаболічний, компенсований Metabolic acidosis, compensated

Ацидоз метаболічний, декомпенсований Metabolic acidosis, decompensated

58 / 200
У хворого, що страждає на порушення мозкового кровообігу, встановлено порушення функції лімбічної системи. Порушення кровопостачання у якій артерії мозку викликало ці симптоми? In a patient suffering from a cerebral blood circulation disorder, a limbic system dysfunction was diagnosed. Blood supply disorder in which brain artery caused these symptoms?

Середня мозкова Average cerebral

Хребтова Khrebtova

Передня мозкова Forebrain

Задня мозкова Hindbrain

Передня ворсинчаста Front nap

59 / 200
Хворому встановлений діагноз ураження голівки стегна ішемічного походження. Яка артерія ушкоджена? The patient has been diagnosed with a lesion of the femoral head of ischemic origin. Which artery is damaged?

Ramus acetabularum A.obturatoriae Branch of dishes A.obturatoria

Arteria profunda femoris Deep femoral artery

Arteria umbilicalis Umbilical artery

Arteria femoralis Femoral artery

Arteria illiaca externa External iliac artery

60 / 200
В психіатричну клініку доставлений хворий 40-ка років у стані збудження, агресії, марення. Який препарат слід ввести хворому? A 40-year-old patient was brought to the psychiatric clinic in a state of excitement, aggression, delirium. What drug should be administered to the patient?

Седуксен Седуксен

Настоянку валеріани Valerian Tincture

Натрію бромід Sodium Bromide

Резерпін Backup

Аміназин Aminazine

61 / 200
У нейрохірургічне відділення надійшов 54-річний чоловік із скаргами на відсутність чутливості шкіри нижньої повіки, латеральної поверхні носа, верхньої губи. Лікар при огляді встановив запалення другої гілки трійчастого нерва. Через який отвір виходить із черепа ця гілка? A 54-year-old man was admitted to the neurosurgery department with complaints of lack of sensitivity of the skin of the lower eyelid, the lateral surface of the nose, and the upper lip. During the examination, the doctor diagnosed inflammation of the second branch of the trigeminal nerve . Through which hole does this branch leave the skull?

Круглий отвір Round hole

Овальний отвір Oval hole

Остистий отвір Prickly Hole

Рваний отвір Torn Hole

Верхня очноямкова щілина Upper orbital fissure

62 / 200
При бактеріологічному дослідженні проб сметани виділені ізольовані культури S.aureus. Як довести етіологічне значення ізольованої культури S.aureus як збудника харчового отруєння, яке виникло серед групи споживачів сметани? During the bacteriological examination of sour cream samples, isolated cultures of S.aureus were isolated. How to prove the etiological significance of the isolated culture of S.aureus as the causative agent of food poisoning that occurred among a group of sour cream consumers?'

Визначення лецитиназної активності Determination of lecithinase activity

Визначення плазмокоагулазної активності Determination of plasma coagulase activity

Визначення цукролітичних властивостей Determination of sucrolytic properties

Виявлення ентеротоксину Enterotoxin Detection

Визначення гемотоксинів Determination of hemotoxins

63 / 200
У дорослої людини за добу виділяється 20 л сечі з низькою відносною щільністю. Найбільш імовірною причиною цього є дефіцит в організмі: An adult produces 20 liters of urine per day with a low relative density. The most likely reason for this is a deficiency in the body:

Натрійуретичного фактора Natriuretic factor

Реніну Renin

Альдостерону Aldosterone

Вазопресину Vasopressin

Паратгормону Parathormone

64 / 200
У чоловіка 28-ми років при гістологічному дослідженні шийного лімфовузла виявлено: порушення малюнка внаслідок розростання епітеліоїдних, лімфоїдних клітин і макрофагів з ядрами у вигляді підкови, в центрі деяких скупчень клітин - безструктурні ділянки блідо-рожевого кольору з уламками ядер. Для якого захворювання характерні такі зміни? In a 28-year-old man, a histological examination of a cervical lymph node revealed: a pattern disturbance due to the growth of epithelioid, lymphoid cells and macrophages with horseshoe-shaped nuclei, in the center of some clusters cells - structureless areas of pale pink color with fragments of nuclei. What disease is characterized by such changes?

Сифіліс Syphilis

Актиномікоз Actinomycosis

Туберкульоз Tuberculosis

Метастаз пухлини Tumor metastasis

Лімфогрануломатоз Lymphogranulomatosis

65 / 200
У хворого з жовтяницею встановлено: підвищення у плазмi крові вмісту загального білірубіну за рахунок непрямого (вільного), в калі та сечі - високий вміст стеркобіліну, рівень прямого (зв’язаного) білірубіну в плазмі крові в межах норми. Про який вид жовтяниці можна думати? In a patient with jaundice, it was established: an increase in the content of total bilirubin in the blood plasma due to indirect (free), high content of stercobilin in feces and urine, level of direct (with bilirubin in the blood plasma is within the normal range. What type of jaundice can you think about?

Хвороба Жильбера Gilbert's disease

Механічна Mechanical

Жовтяниця немовлят Infant Jaundice

Гемолітична Hemolytic

Паренхіматозна (печінкова) Parenchymatous (hepatic)

66 / 200
При штовханні штанги спортсмен закидає голову назад для максимального підвищення тонусу м’язів-розгиначів верхніх кінцівок. Де розташовані центри рефлексів, які при цьому виникають? When pushing the barbell, the athlete throws his head back to maximize the tone of the extensor muscles of the upper limbs. Where are the centers of reflexes that arise?

Ядра Дейтерса Deiters Kernels

Базальні ганглії Basal ganglia

Рухова кора Motor cortex

Червоні ядра Red kernels

Спинний мозок Spinal cord

67 / 200
Знешкодження ксенобіотиків (лікарських засобів, епоксидів, ареноксидів, альдегідів, нітропохідних тощо) та ендогенних метаболітів (естрадіолу, простагландинів, лейкотрієнів) відбувається в печінці шляхом їх кон’югації з: Detoxification of xenobiotics (drugs, epoxides, arenoxides, aldehydes, nitroderivatives, etc.) and endogenous metabolites (estradiol, prostaglandins, leukotrienes) occurs in the liver through their conjugation with:

Глутатіоном Glutathione

S-Аденозилметіоніном S-Adenosylmethionine

Фосфоаденозином Phosphoadenosine

Гліцином Glycinema

Аспарагіновою кислотою Aspartic acid

68 / 200
При вивченні родоводу сім’ї, в якій спостерігається гіпертрихоз (надмірне оволосіння вушних раковин), виявлена ознака трапляється в усіх поколіннях тільки у чоловіків і успадковується від батька до сина. Визначте тип успадкування гіпертрихозу: When studying the genealogy of a family in which hypertrichosis (excessive hairiness of the auricles) is observed, the identified trait occurs in all generations only in men and is inherited from father to son Determine the type of inheritance of hypertrichosis:

Аутосомно-домінантний Autosomal dominant

Аутосомно-рецесивний Autosomal recessive

Зчеплений з Х-хромосомою домінантний X-linked dominant

Зчеплений з Х-хромосомою рецесивний X-linked recessive

Зчеплений з Y-хромосомою Linked to Y-chromosome

69 / 200
У спортсмена після інтенсивного тренування відзначається значне зниження тонусу судин у ділянці працюючих м’язів. Причиною розвитку такого ефекту є накопичення у працюючих тканинах: After intensive training, an athlete has a significant decrease in vascular tone in the area of working muscles. The reason for the development of this effect is accumulation in working tissues:

Натрійуретичного гормону Natriuretic hormone

Серотоніну Serotonin

Ренін-ангіотензину Renin-angiotensin

Гістаміну Histamine

Метаболітів Metabolites

70 / 200
Внаслідок активації іонних каналів зовнішньої мембрани збудливої клітини значно збільшився її потенціал спокою. Які канали були активовані? As a result of the activation of the ion channels of the outer membrane of the excitable cell, its resting potential increased significantly. Which channels were activated?

Натрієві та кальцієві Sodium and calcium

Калієві Potassium

Повільні кальцієві Slow calcium

Натрієві Sodium

Швидкі кальцієві Fast calcium

71 / 200
Чоловіку 40-ка років за вимогою діагностичних тестів зробили лімфографію органів грудної порожнини. Хірург встановив, що пухлина вразила орган, з лімфатичних судин якого лімфа безпосередньо переходить в грудну протоку. Який це орган? A 40-year-old man underwent a lymphography of the organs of the chest cavity at the request of diagnostic tests. The surgeon established that the tumor affected the organ from whose lymphatic vessels the lymph directly passes into the thoracic duct . What organ is this?

Осердя Hearts

Трахея Trachea

Серце Heart

Лівий головний бронх Left main bronchus

Стравохід Esophagus

72 / 200
У хірургічне відділення лікарні надійшла хвора з явищами гострого панкреатиту: блювання, пронос, сильний оперізуючий біль, слабкість, гіпотензія, зневодненням організму. Який препарат з антиферментною активністю показаний хворому? A patient was admitted to the surgical department of the hospital with symptoms of acute pancreatitis: vomiting, diarrhea, severe girdling pain, weakness, hypotension, dehydration. What drug with anti-enzyme activity is indicated for the patient ?

Атропіну сульфат Atropine sulfate

Адреналін Adrenaline

Натрію гідрокарбонат Sodium bicarbonate

Контрикал Kontrykal

Анальгін Analgin

73 / 200
На препараті нирки розрізняються нефрони, які лежать на межі між кірковою та мозковою речовиною, мають однаковий діаметр приносних і виносних артеріол. Назвіть, яка функція буде порушена при їхньому пошкодженні? Nephrons are distinguished on the kidney preparation, which lie on the border between the cortex and the medulla, have the same diameter of afferent and efferent arterioles. Name which function will be impaired when they are damaged ?

Шунтування крові при інтенсивному кровообігу Blood shunting with intensive blood circulation

Синтез реніну Synthesis of renin

Синтез еритропоетину Erythropoietin Synthesis

Активність натрієвого рецептора Natrium receptor activity

Синтез простагландинів Synthesis of prostaglandins

74 / 200
Під час гістологічного дослідження стулок мітрального клапана серця жінки 30-ти років було встановлено, що ендотеліальні клітини вогнищево десквамовані, в цих ділянках на поверхні стулки розташовані дрібні тромботичні нашарування, сполучна тканина стулки з явищами мукоїдного набухання з ділянками склерозу та васкуляризації. Діагностуйте вид клапанного ендокардиту: During a histological examination of the leaflets of the mitral valve heart of a 30-year-old woman, it was established that the endothelial cells are focally desquamated, in these areas there are small thrombotic layers on the surface of the leaflet, leaflet connective tissue with mucoid swelling phenomena with areas of sclerosis and vascularization. Diagnose the type of valvular endocarditis:

Поліпозно-виразковий Polypo-ulcerative

Гострий бородавчастий Sharp warty

Фібропластичний Fibroplastic

Поворотньо-бородавчастий Rotary-warty

Дифузний Diffuse

75 / 200
Хворий переніс інсульт. Який з наведених препаратів слід включити до комплексної терапії з метою покращення кровообігу та метаболізму головного мозку? The patient suffered a stroke. Which of the following drugs should be included in complex therapy to improve blood circulation and brain metabolism?

Амітриптилін Amitriptyline

Камфора Camphora

Седуксен Седуксен

Феназепам Phenazepam

Пірацетам Piracetam

76 / 200
Відзначте концентрацію етилового спирту, що має найбільш активну протимікробну дію за наявністю білку у середовищі: Mark the concentration of ethyl alcohol that has the most active antimicrobial effect according to the presence of protein in the environment:

40% 40%

15% 15%

60% 60%

96% 96%

70% 70%

77 / 200
У хворого 40-ка років при прогресуванні стафілококового гнійного періодонтиту виникло гнійне запалення кістково-мозкових просторів альвеолярного відростка, а потім тіла нижньої щелепи. Мікроскопічно кісткові балки витончені, вогнища некрозу, кісткові секвестри, оточені сполучнотканинною капсулою. Який найбільш імовірний діагноз? A 40-year-old patient developed purulent inflammation of the bone-marrow spaces of the alveolar process and then the body of the lower jaw during the progression of staphylococcal purulent periodontitis. Microscopically, the bone beams are thin, foci necrosis, bone sequestrations surrounded by a connective tissue capsule. What is the most likely diagnosis?

Хронічний фіброзний періостит Chronic fibrous periostitis

Гострий остеомієліт Acute osteomyelitis

Хронічний остеомієліт Chronic osteomyelitis

Гнійний періостит Suppurative periostitis

Пародонтома Periodontoma

78 / 200
Група чоловіків звернулася до лікаря зі скаргами на підвищення температури, головний біль, набряки повік та обличчя, біль у м’язах. З анамнезу: всі вони мисливці і часто вживають в їжу м’ясо диких тварин. Який найбільш імовірний діагноз? A group of men went to the doctor with complaints of fever, headache, swelling of the eyelids and face, muscle pain. From the anamnesis: they are all hunters and often eat the meat of wild animals. What is the most likely diagnosis?

Теніарінхоз Taeniarhynchosis

Філяріатоз Filariasis

Цистицеркоз Cysticercosis

Трихінельоз Trichinellosis

Теніоз Taeniosis

79 / 200
Під час операції видалення матки з яєчниками і матковими трубами лікар перев’язує зв’язку, що підвішує яєчник. Які судини перев’язав лікар в цій зв’язці? During the operation to remove the uterus with ovaries and fallopian tubes, the doctor ligates the ligament that suspends the ovary. What vessels did the doctor ligate in this ligament ?

Внутрішня клубова вена Internal iliac vein

Яєчникові артерія і вена Ovarian artery and vein

Маткові артерія і вена Uterine artery and vein

Внутрішня клубова артерія Internal iliac artery

Трубні артерія і вена Tubes artery and vein

80 / 200
При проведенні операції на тонкій кишці лікар виявив ділянку слизової оболонки, де на фоні колових складок була присутня поздовжня складка. Який відділ тонкої кишки має таку будову? When performing an operation on the small intestine, the doctor found a section of the mucous membrane where a longitudinal fold was present against the background of circular folds. Which part of the small intestine has this structure?

Дистальний вiддiл ileum Distal division of ileum

Початковий eiddrn jejunum Initial eiddrn jejunum

Pars ascendens duodeni Ascending part of twelve

Pars descendens duodeni Descending part of the duodenum

Pars horizontalis duodeni Pars horizontalis duodeni

81 / 200
Під час гістологічного дослідження легень хворого, що помер від серцевої недостатності, виявлені вогнища запалення з заповненням альвеол рідиною, забарвленою у блідорожевий колір, місцями з наявністю тонких рожевих ниток, що утворюють дрібнопетлисту сітку з невеликою кількістю лімфоцитів. Який характер ексудату у легенях? During the histological examination of the lungs of a patient who died of heart failure, foci of inflammation were found with the alveoli being filled with liquid colored in a pale pink color, in places with the presence of thin pink threads, which form a small looped network with a small number of lymphocytes. What is the nature of the exudate in the lungs?

Серозний Serious

Геморагічний Hemorrhagic

Серозно-фібринозний Serous-fibrinous

Фібринозний Fibrinous

Гнійний Suppurative

82 / 200
У хворого нормально забарвлений кал, у складі якого з находиться велика кількість вільних жирних кислот. Причиною цього є порушення наступного процесу: The patient has normally colored stool, which contains a large amount of free fatty acids. The reason for this is a violation of the following process:

Жовчоутворення Cholelithiasis

Всмоктування жирів Lip suction

Секреція ліпаз Secretion of lipases

Гідроліз жирів Hydrolysis of fats

Жовчовиділення Bile

83 / 200
При термометрії встановлено, що температура відкритих ділянок шкіри на 1-1,5° нижче за температуру поруч розташованих ділянок, закритих одягом з натуральних тканин. Причиною цього є те, що одяг, перш за все, зменшує тепловіддачу таким шляхом: During thermometry, it was established that the temperature of open areas of the skin is 1-1.5° lower than the temperature of adjacent areas covered by clothes made of natural fabrics. The reason for this is that , that clothing, first of all, reduces heat transfer in the following way:

Випаровування Evaporation

Конвекція Convection

Радіація Radiation

Проведення Execution

- -

84 / 200
У хворого діагностовано алкаптонурію. Вкажіть фермент, дефект якого є причиною цієї патології: The patient is diagnosed with alkaptonuria. Specify the enzyme whose defect is the cause of this pathology:

Піруватдегідрогеназа Pyruvate dehydrogenase

Оксидаза гомогентизинової кислоти Homogentisic acid oxidase

Фенілаланінгідроксилаза Phenylalanine hydroxylase

ДОФА-декарбоксилаза DOPA Decarboxylase

Глутаматдегідрогеназа Glutamate dehydrogenase

85 / 200
На препараті м’якої мозкової оболонки виявляється судина, у стінці якої відсутня середня оболонка, зовнішня оболонка зрощена з оточуючою тканиною, внутрішня оболонка побудована із базальної мембрани та ендотелію. Що це за судина? On the preparation of the medulla, a vessel is found in the wall of which there is no middle membrane, the outer membrane is fused with the surrounding tissue, the inner membrane is made of basement membrane and endothelium. What kind of vessel is this?

Артерія м’язового типу Muscular artery

Вена м’язового типу зі слабким розвитком м’язових елементів Muscular type vein with weak development of muscular elements

Артеріола Arteriole

Вена волокнистого типу Fibrous type vein

Артерія змішаного типу Mixed type artery

86 / 200
На розтині тіла померлого від інтоксикації в тонкій кишці знайдено набряк групових лімфатичних фолікулів, виступаючих над поверхнею слизової оболонки у вигляді м’якоеластичних бляшок з нерівною поверхнею у вигляді борозен і звивин, що нагадують поверхню мозку. Який діагноз найбільш імовірний? At the autopsy of the body of the deceased from intoxication, swelling of group lymphatic follicles was found in the small intestine, protruding above the surface of the mucous membrane in the form of soft elastic plaques with an uneven surface in the form of furrows and gyri resembling the surface of the brain. What is the most likely diagnosis?

Сальмонельоз Salmonellosis

Гострий ентерит Acute enteritis

Холера Cholera

Дизентерія Dysentery

Черевний тиф Typhoid

87 / 200
При виконуванні вправ на колоді гімнастка втратила рівновагу і впала. Із збудження, перш за все, яких рецепторів розпочнуться рефлекси, що забезпечать відновлення порушеної пози? While performing exercises on the floor, the gymnast lost her balance and fell. From excitement, first of all, which receptors will trigger the reflexes that will ensure the restoration of the broken posture?

Отолітові вестибулорецептори Otolith vestibuloreceptors

Ампулярні вестибулорецептори Ampullary vestibuloreceptors

Рецептори завитки Curl Receptors

Вестибулорецептори Vestibuloreceptors

Пропріорецептори Proprioreceptors

88 / 200
Під час обстеження хворої виникла підозра на наявність гнійного випоту в прямокишково-матковому заглибленні. Через яке анатомічне утворення найкраще пропунктувати дане заглиблення? During the examination of the patient, there was a suspicion of the presence of purulent effusion in the recto-uterine depression. Through which anatomical formation is the best way to puncture this depression?

Діафрагма тазу Pelvic diaphragm

Переднє склепіння піхви Anterior vaginal vault

Заднє склепіння піхви Posterior vaginal vault

Ампула прямої кишки Ampoule of the rectum

Передня стінка піхви Anterior vaginal wall

89 / 200
Хворий 35-ти років звернувся зі скаргами на біль та набряк в ділянці дна ротової порожнини. Діагностовано запальний процес у ділянці вивідної протоки піднижньощелепної слинної залози. Куди відкривається ця протока? A 35-year-old patient complained of pain and swelling in the area of the floor of the oral cavity. An inflammatory process was diagnosed in the area of the excretory duct of the submandibular salivary gland. Where does this duct open ?

Recesus gingivalis Recesus gingivalis

Foramen caecum linguae Language blind hole

Plica fimbriata Fringed fold

Caruncula sublingualis Sublingual caruncle

Vestibulum oris The lobby of the mouth

90 / 200
До хірурга звернувся чоловік 60-ти років, що тривалий час хворіє на цукровий діабет. Об’єктивно: тканини правої стопи чорного кольору, щільні, з чіткими краями. Який діагноз поставив хірург? A 60-year-old man who has been suffering from diabetes for a long time turned to the surgeon. Objectively: the tissues of the right foot are black, dense, with clear edges. What diagnosis did the surgeon make?

Пролежень Decubitus

Суха гангрена Dry Gangrene

Газова гангрена Gas Gangrene

Волога гангрена Wet gangrene

Трофічна виразка Tropical ulcer

91 / 200
Запалення характеризується розширенням кровоносних судин на ділянці пошкодження, зменшенням кровообігу, підвищенням проникливості стінки судин. Яким з нижче наведених клітин належить головна роль в цьому? Inflammation is characterized by the expansion of blood vessels in the area of damage, a decrease in blood circulation, and an increase in the permeability of the vessel wall. Which of the following cells plays the main role in this?

Фібробласти Fibroblasts

Еозинофіли Eosinophils

Плазмоцити Plasmocytes

Макрофаги Macrophages

Тканинні базофіли Tissue basophils

92 / 200
Хворий після перенесеного епідемічного паротиту схуднув, постійно відчуває спрагу, п’є багато води, відмічає часте сечовиділення, підвищений апетит, шкірний свербіж, слабкість, фурункульоз. У крові: глюкоза - 16 ммоль/л, кетонових тіл - 100 мкмоль/л; глюкозурія. Яке захворювання розвинулось у пацієнта? After suffering from epidemic mumps, the patient has lost weight, is constantly thirsty, drinks a lot of water, notes frequent urination, increased appetite, skin itching, weakness, furunculosis. In the blood : glucose - 16 mmol/l, ketone bodies - 100 μmol/l; glucosuria. What disease did the patient develop?

Інсулінонезалежний цукровий діабет Insulin-dependent diabetes

Стероїдний діабет Steroid diabetes

Інсулінозалежний цукровий діабет Insulin-dependent diabetes

Цукровий діабет недостатнього харчування Diabetes undernutrition

Нецукровий діабет Diabetes insipidus

93 / 200
На гістологічне дослідження надіслано видалений червоподібний відросток. Розміри його збільшені, серозна оболонка тьмяна, повнокровна, вкрита плівками фібрину, стінки стовщені, на розрізі із просвіту виділяється гній. При мікроскопічному дослідженні спостерігається повнокров’я судин, набряк всіх шарів і дифузна інфільтрація їх лейкоцитами. Назвіть форму гострого апендициту: The removed appendix was sent for histological examination. Its dimensions are increased, the serous membrane is dull, full of blood, covered with fibrin films, the walls are thickened, pus is released from the lumen on the cut. Microscopic examination shows fullness of blood vessels, swelling of all layers and diffuse infiltration of leukocytes. Name the form of acute appendicitis:

Простий Simple

Флегмонозний Phlegmonous

Поверховий Floor

Апостематозний Apostematous

Гангренозний Gangrenous

94 / 200
У хворого 23-х років в результаті черепно-мозкової травми виник набряк мозку. Який механізм пошкодження клітин безпосередньо призвів до набряку мозку? A 23-year-old patient developed brain swelling as a result of a brain injury. What mechanism of cell damage directly led to brain swelling?

Ацидотичний Acidotic

Електролітно-осмотичний Electrolyte-osmotic

Ліпідний Lipid

Кальцієвий Calcium

Протеїновий Protein

95 / 200
У хворого, який скаржився на біль у ділянці лівої лопатки, був діагностований інфаркт міокарду. Назвіть вид болю у хворого? A patient who complained of pain in the area of the left scapula was diagnosed with a myocardial infarction. Name the type of pain in the patient?

Перший (протопатичний) First (protopathic)

Вісцеральний Visceral

Іррадіюючий (відбитий) Irradiating (reflected)

Фантомний Phantomnius

Другий (епікритичний) Second (Epicritic)

96 / 200
Хвора 38-ми років надійшла до реанімаційного відділення в несвідомому стані. Рефлекси відсутні. Цукор крові -2,1 ммоль/л. В анамнезі - цукровий діабет з 18-ти років. Яка кома має місце у хворої? A 38-year-old patient came to the intensive care unit in an unconscious state. There are no reflexes. Blood sugar -2.1 mmol/l. History of diabetes since 18 'You are years old. What kind of coma does the patient have?

Кетоацидотична Ketoacidotic

Гіпоглікемічна Hypoglycemic

Лактацидемічна Lactacidemic

Гіперосмолярна Hyperosmolar

Гіперглікемічна Hyperglycemic

97 / 200
До фібрилярних елементів сполучної тканини належать колаген, еластин та ретикулін. Вкажіть амінокислоту, яка входить тільки до складу колагену і визначення якої в біологічних рідинах використовується для діагностики захворювань сполучної тканини: Fibrillar elements of connective tissue include collagen, elastin, and reticulin. Name the amino acid that is only part of collagen and whose determination in biological fluids is used to diagnose connective tissue diseases :

Лізин Lysine

Пролін Пролін

Гліцин Glycine

Гідроксипролін Hydroxyproline

Фенілаланін Phenylalanine

98 / 200
Пацієнт через 15 діб після повернення з багатомісячного плавання в районах Середземномор’я та Західної Африки відчув слабкість, головний біль, періодичні підвищення температури. Лікар запідозрив у хворого малярію. Який із перерахованих методів є найбільш адекватним в діагностиці даного захворювання? 15 days after returning from months of sailing in the Mediterranean and West Africa, the patient experienced weakness, headache, and periodic fever. The doctor suspected the patient of malaria. Which of the listed methods is the most adequate in the diagnosis of this disease?

Мікробіологічний Microbiological

Мікроскопічний Microscopic

Біологічний Biological

Серологічний Serological

Алергічний Allergic

99 / 200
У вагітної жінки взяли кров для підтвердження клінічного діагнозу 'токсоплазмоз'. Яка із перерахованих серологічних реакцій має діагностичне значення? Blood was taken from a pregnant woman to confirm the clinical diagnosis of 'toxoplasmosis'. Which of the listed serological reactions has diagnostic significance?

Реакція аглютинації Agglutination reaction

Реакція гальмування гемаглютинації Hemagglutination inhibition reaction

Реакція гемадсорбції Hemadsorption reaction

Реакція зв’язування комплементу Complement binding reaction

Реакція нейтралізації Neutralization reaction

100 / 200
У здорових батьків, спадковість яких не обтяжена, народилась дитина з чисельними вадами розвитку. Цитогенетичний аналіз виявив в соматичних клітинах дитини трисомію за 13-ю хромосомою (синдром Патау). З яким явищем пов’язане народження такої дитини? A child with multiple developmental disabilities was born to healthy parents whose heredity is not burdened. Cytogenetic analysis revealed trisomy on the 13th chromosome (Patau syndrome) in the child's somatic cells What phenomenon is associated with the birth of such a child?

Рецесивна мутація Recessive mutation

Соматична мутація Somatic mutation

Домінантна мутація Dominant mutation

Хромосомна мутація Chromosome mutation

Порушення гаметогенезу Disruption of gametogenesis

101 / 200
Для запобігання нападів гострого панкреатиту лікар призначив трасілол (контрікал, гордокс), який є інгібітором: To prevent attacks of acute pancreatitis, the doctor prescribed trasilol (contrikal, gordox), which is an inhibitor:

Карбоксипептидази Carboxypeptidase

Хімотрипсину Chemotrypsin

Еластази Elastasi

Гастриксину Gastrixin

Трипсину Trypsin

102 / 200
Після травми на рентгенограмі постраждалого визначається перелом плеча у ділянці гребеня великого горбика. Функція якого м’яза буде у цьому випадку порушена? After the injury, the x-ray of the victim shows a fracture of the shoulder in the area of the crest of the greater tubercle. Which muscle function will be impaired in this case?

Підключичний Subkey

Великий грудний Big Chest

Малий грудний Small chest

Дельтоподібний Delta

Передній зубчастий Front gear

103 / 200
Хворий помер від прогресуючої серцевої недостатності. На розтині серце розширене у поперечнику, мляве, м’яз на розрізі нерівномірного кровонаповнення, пістрявий, при гістологічному дослідженні у міокарді повнокров’я, у стромі лімфогістіоцитарні інфільтрати, що розсувають кардіоміоцити. Виявлені морфологічні зміни свідчать про: The patient died of progressive heart failure. At autopsy, the heart is dilated in cross-section, lethargic, the muscle on the cross-section has uneven blood supply, mottled, during histological examination, the myocardium is full of blood' i, in the stroma, there are lymphohistiocytic infiltrates, displacing cardiomyocytes. The detected morphological changes indicate:

Жирову дистрофії міокарда Fatty myocardial dystrophy

Інфаркт міокарда Myocardial infarction

Негнійний проміжний міокардит Nonpurulent interstitial myocarditis

Венозне повнокрів’я Venous whole blood

Кардіосклероз Cardiosclerosis

104 / 200
В експерименті показано, що при саркомі Ієнсена споживання глюкози з привідної до пухлини артерії значно збільшується, має місце також приріст вмісту молочної кислоти у відвідній вені. Про що свідчить дане явище? In the experiment, it was shown that with Jensen's sarcoma, the consumption of glucose from the artery leading to the tumor increases significantly, there is also an increase in the content of lactic acid in the efferent vein. This is evidenced by phenomenon?

Зменшення анаеробного гліколізу Reduction of anaerobic glycolysis

Посилення окисних процесів Intensification of oxidative processes

Зменшення окисних процесів Reduction of oxidative processes

Посилення анаеробного гліколізу Enhancement of anaerobic glycolysis

Посилення окиснення білків Increased protein oxidation

105 / 200
У хворого 23-х років після перенесеної ангіни розвинувся сечовий синдром (гематурія, протеїнурія, лейкоцитурія). У пункційній біопсії нирок виявлена картина інтракапілярного проліферативного гломерулонефриту, а електронномікроскопічно виявлені великі субепітеліальні депозити. Який патогенез цього захворювання? A 23-year-old patient developed a urinary syndrome (hematuria, proteinuria, leukocyturia) after a sore throat. A puncture biopsy of the kidneys revealed a picture of intracapillary proliferative glomerulonephritis, and electron microscopic large subepithelial deposits. What is the pathogenesis of this disease?

Імунокомплексний механізм Immune complex mechanism

Атопія Atopy

Клітинно обумовлений цитоліз Cell-induced cytolysis

Цитотоксична, цитолітична дія антитіл Cytotoxic, cytolytic effect of antibodies

Грануломатоз Granulomatosis

106 / 200
У 29-річної породіллі на 3-й день після пологів виник дифузний набряк правої грудної залози, болючість при пальпації, гіперемія шкіри у вказаній ділянці, підвищення температури тіла до 38oC. При гістологічному дослідженні тканини залози виявлено: в стромі - дифузний клітинний інфільтрат, який складається з великої кількості нейтрофільних лейкоцитів, інтерстиційний набряк, гіперемія судин. Діагностуйте захворювання: On the 3rd day after giving birth, a 29-year-old woman in labor developed diffuse swelling of the right breast, tenderness on palpation, hyperemia of the skin in the indicated area, an increase in body temperature to 38oC. Histological examination of the gland tissue revealed: in the stroma - a diffuse cellular infiltrate consisting of a large number of neutrophilic leukocytes, interstitial edema, vascular hyperemia. Diagnose the disease:

Гострий апостематозний мастит Acute apostematous mastitis

Гострий флегмонозний мастит Acute phlegmonous mastitis

Хронічний гнійний мастит Chronic purulent mastitis

Хронічний продуктивний мастит Chronic productive mastitis

Гострий серозний мастит Acute serous mastitis

107 / 200
У хворого на хронічну серцеву недостатність, незважаючи на терапію кардіотонічними засобами і тіазидовим діуретиком, зберігаються набряки і виникла загроза асциту. Який препарат слід призначити для підсилення діуретичного ефекту застосованих ліків? In a patient with chronic heart failure, despite therapy with cardiotonic agents and a thiazide diuretic, edema persists and there is a threat of ascites. What drug should be prescribed to enhance the diuretic effect of the drugs used ?

Фуросемід Furosemide

Амілорид Amiloride

Манітол Mannitol

Клопамід Clopamide

Спіронолактон Spironolactone

108 / 200
До лікарні госпіталізовано хворого з підозрою на черевний тиф. Який матеріал необхідно взяти у нього з метою ранньої діагностики цього захворювання? A patient with suspected typhoid fever was admitted to the hospital. What material should be taken from him for the purpose of early diagnosis of this disease?

Кров Кров

Фекалії Feces

Жовч Bile

Сеча Сеча

Кістковий мозок Bone marrow

109 / 200
У чоловіка 30-ти років перед операцією визначили групову належність крові. Кров резуспозитивна. Реакцію аглютинації еритроцитів не викликали стандартні сироватки груп 0αβ (I), Аβ (II), Вα (III). Досліджувана кров належить до групи: The blood group of a 30-year-old man was determined before the operation. The blood is Rh-positive. The erythrocyte agglutination reaction was not caused by standard sera of groups 0αβ (I), Aβ (II) , Bα (III). The studied blood belongs to the group:

Вα (III) Вα (III)

- -

Аβ (II) Аβ (II)

0αβ (I) 0αβ (I)

АВ (IV) АВ (IV)

110 / 200
У людини збільшений вміст іонів кальцію в плазмі крові, зменшений - у кістках. Надмірна секреція якого гормону може спричинити такі зміни? A person has an increased content of calcium ions in the blood plasma, decreased - in the bones. Excessive secretion of which hormone can cause such changes?

Трийодтиронін Triiodothyronine

Тиреокальцитонін Thyrocalcitonin

Тироксин Thyroxine

Паратгормон Parathyroid hormone

Альдостерон Aldosterone

111 / 200
У студента, який складає іспит, вміст глюкози у плазмі крові складає 8 ммоль/л. Збільшена секреція якого з наведених гормонів сприяє розвитку гіперглікемії у студента? A student taking an exam has a blood glucose content of 8 mmol/l. Increased secretion of which of the following hormones contributes to the development of hyperglycemia in a student?

Інсулін Insulin

Трийодтиронін Triiodothyronine

Альдостерон Aldosterone

Глюкагон Glucagon

Тироксин Thyroxine

112 / 200
У хворого з неврологічними порушеннями діагностована пухлина головного мозку. Під час операції видалена пухлина, що має вид щільного вузла, пов’язаного з твердою мозковою оболонкою. Гістологічно пухлина побудована з ендотеліоподібних клітин, тісно прилеглих одна до одної. Який найбільш імовірний діагноз? A patient with neurological disorders was diagnosed with a brain tumor. During the operation, the tumor was removed, which has the appearance of a dense node connected to the dura mater. Histologically, the tumor was constructed from endothelial-like cells closely adjacent to each other. What is the most likely diagnosis?

Нейробластома Neuroblastoma

Менінгеальна саркома Meningeal sarcoma

Менінгеома Meningioma

Гліобластома Glioblastoma

Астроцитома Astrocytoma

113 / 200
На аутопсії жінки, що хворіла на хронічну дизентерію, при гістологічному дослідженні внутрішніх органів у стромі та паренхімі міокарда, нирок, у слизовій оболонці шлунка, у сполучній тканині легень виявлені аморфні відкладання фіолетового кольору, що дають позитивну реакцію за Коссом. Яке ускладнення розвинулось у хворої? At the autopsy of a woman suffering from chronic dysentery, during histological examination of internal organs in the stroma and parenchyma of the myocardium, kidneys, in the mucous membrane of the stomach, in the connective tissue of the lungs, amorphous deposits of purple color, which give a positive reaction according to Koss. What complication developed in the patient?

Метаболічне звапніння Metabolic calcification

Метастатичне звапніння Metastatic calcification

Дистрофічне звапніння Dystrophic calcification

Амілоїдоз Amyloidosis

Гіаліноз Hyalinosis

114 / 200
Медсестра зі стажем роботи 10 років захворіла на контактний дерматит верхніх кінцівок. До якого типу імунної патології відноситься це захворювання? A nurse with 10 years of experience developed contact dermatitis of the upper extremities. What type of immune pathology does this disease belong to?

Первинний імунодефіцит Primary immunodeficiency

Т-клітинний імунодефіцит T-cell immunodeficiency

Алергічна реакція сповільненого типу Delayed allergic reaction

В-клітинний імунодефіцит B-cell immunodeficiency

Алергічна реакція негайного типу Allergic reaction of immediate type

115 / 200
У хворого, прооперованого з приводу 'гострого живота', сеча коричневого кольору, кількість індикану в сечі вище 93 ммоль/добу. Про що це свідчить? A patient operated on for an 'acute abdomen' has brown urine, the amount of indican in the urine is higher than 93 mmol/day. What does this indicate?

Зниження інтенсивності знезараження амоніаку Decreasing the intensity of ammonia disinfection

Порушення фільтраційної здатності нирок Disruption of the filtration capacity of the kidneys

Збільшення швидкості окисного дезамінування ароматичних амінокислот Increasing rate of oxidative deamination of aromatic amino acids

Зниження активності ферментів орнітинового циклу Decreased activity of enzymes of the ornithine cycle

Збільшення інтенсивності гниття білків у кишечнику Increasing intensity of protein decay in intestines

116 / 200
У реанімаційному відділенні знаходиться хворий у коматозному стані. При дослідженні крові відзначено збільшення концентрації іонів K+ і зменшення - Ca++, ацидоз, збільшення рівнів сечовини, сечової кислоти. Який вид коми за етіологією найбільш імовірний? There is a patient in a comatose state in the intensive care unit. Blood tests showed an increase in the concentration of K+ ions and a decrease in - Ca++, acidosis, increased levels of urea, uric acid. What type coma is the most probable etiology?

Діабетична Diabetic

Гіпоглікемічна Hypoglycemic

Ниркова Kidney

Печінкова Hepatic

Нейрогенна Neurogenic

117 / 200
Жінка в період вагітності тривалий час безконтрольно приймала хіміотерапевтичний препарат. Через деякий час у неї погіршився апетит, з’явились нудота, пронос. З часом виникла жовтяниця. У новонародженого відмічено порушення росту кісток. Який препарат з групи тетрацикліну приймала жінка? During pregnancy, a woman took a chemotherapeutic drug uncontrollably for a long time. After a while, her appetite worsened, nausea, diarrhea appeared. Over time, jaundice developed. The newborn a violation of bone growth was noted. Which drug from the tetracycline group did the woman take?

Бензилпеніциліну натрiєва сіль Benzylpenicillin sodium salt

Доксицикліну гідрохлорид Doxycycline hydrochloride

Азитроміцин Azithromycin

Бісєптол Biseptol

Ципрофлоксацин Ciprofloxacin

118 / 200
Під час розтину тіла дитини, яка померла при ознаках асфіксії, були виявлені в трахеї і головних бронхах сіруватого кольору плівки, які вільно лежали у просвіті дихальних шляхів, нагадуючи їх зліпки. Вкажіть вид запалення: During the autopsy of the body of a child who died with signs of asphyxia, grayish-colored films were found in the trachea and main bronchi, which lay freely in the lumen of the respiratory tract, resembling them casts. Specify the type of inflammation:

Дифтеритичне Diphtheritic

Крупозне Крупозне

Гнійне Purulent

Катаральне Cataral

Серозне Serozne

119 / 200
Чоловік 60-ти років скаржиться на біль у суглобах. У сироватці крові пацієнта виявлено підвищення концентрації С-реактивного білку та оксипроліну. Для якого захворювання характерні ці симптоми? A 60-year-old man complains of joint pain. An increase in the concentration of C-reactive protein and oxyproline was detected in the patient's blood serum. What disease are these symptoms characteristic of?

Гепатит Hepatitis

Ревматизм Rheumatism

Жовтяниця Jaundice

Цукровий діабет Diabetes

Подагра Gout

120 / 200
До офтальмолога звернувся пацієнт зі скаргами на різі в очах. При обстеженні встановлена ерозія рогівки - відсутність поверхневого і шипуватого шарів епітелію. Які клітини будуть забезпечувати регенерацію ушкодженого епітелію? A patient turned to an ophthalmologist with complaints of cuts in the eyes. During the examination, corneal erosion was established - the absence of the surface and spiny layers of the epithelium. What cells will ensure the regeneration of the damaged epithelium?

Клітини рогового шару Cells of the stratum corneum

Клітини блискучого шару Shiny Layer Cells

Клітини зернистого шару Cells of the granular layer

Базальні Basal

Клітини поверхневого шару Cells of the surface layer

121 / 200
До інфекційного відділення госпіталізували хворого з ознаками загальної слабкості, сильними головними і м’язовими болями, високою температурою, гіперемією обличчя. Встановлено, що тиждень тому хворий відпочивав біля озера. Лікар запідозрив лептоспіроз. Яким чином лептоспіри могли потрапити до організму хворого? A patient was hospitalized in the infectious department with signs of general weakness, severe headache and muscle pain, high temperature, facial hyperemia. It was established that a week ago the patient was resting near the lake . The doctor suspected leptospirosis. How could leptospires get into the patient's body?

З повітрям With air

З водою With water

З ґрунтом With soil

Через предмети вжитку Because of consumables

З їжею With food

122 / 200
У дорослої людини системний артеріальний тиск знизився з 120/70 до 90/50 мм рт.ст., що викликало рефлекторне звуження судин. У якому з зазначених органів звуження судин буде найменшим? In an adult, the systemic arterial pressure decreased from 120/70 to 90/50 mm Hg, which caused reflex vasoconstriction. In which of the specified organs is the constriction vessels will be the smallest?

Шкіра Skin

Скелетні м’язи Skeletal muscles

Кишечник Intestines

Печінка Liver

Серце Серце

123 / 200
Під час хірургічного втручання на тонкій кишці у людини можлива рефлекторна зупинка серця. Які рецептори в міокарді необхідно заблокувати, щоб попередити зупинку? During surgery on the small intestine, reflex cardiac arrest is possible in a person. What receptors in the myocardium must be blocked to prevent arrest?

H-холінорецептори H-cholinergic receptors

α-адренорецептори α-adrenergic receptors

M-холінорецептори M-cholinergic receptors

Пуринові рецептори Purine receptors

β-адренорецептори β-adrenergic receptors

124 / 200
У лікарню наприкінці робочого дня доставлений робітник 'гарячого' цеху, який скаржиться на головний біль, запаморочення, нудоту, загальну слабкість. Об’єктивно: свідомість збережена, шкірні покриви гіперемовані, сухі, гарячі на дотик. ЧСС- 130/хв. Дихання часте, поверхневе. Яке порушення процесів терморегуляції, найбільш імовірно, виникло у людини в даній ситуації? At the end of the working day, a worker in a 'hot' workshop was taken to the hospital, complaining of headache, dizziness, nausea, general weakness. Objectively: consciousness is preserved, skin the integuments are hyperemic, dry, hot to the touch. Heart rate - 130/min. Breathing is frequent, shallow. What violation of thermoregulation processes most likely occurred in a person in this situation?

Посилення теплопродукції без змін тепловіддачі Increased heat production without changes in heat output

Зниження теплопродукції без змін тепловіддачі Reduction of heat production without changes in heat output

Зниження тепловіддачі Reduction of heat transfer

Посилення тепловіддачі і теплопродукції Increasing heat transfer and heat production

Посилення тепловіддачі і зниження теплопродукції Increasing heat transfer and decreasing heat production

125 / 200
Експериментальне вивчення нового медичного препарату виявило блокуючий ефект на збирання білків-тубулінів, які є основою веретена поділу в клітинах, що діляться. Який етап клітинного циклу порушується цим препаратом? An experimental study of a new medical drug revealed a blocking effect on the assembly of tubulin proteins, which are the basis of the division spindle in dividing cells. What stage of the cell cycle is disrupted by this drug?

Премітотичний період інтерфази Premitotic interphase period

Телофаза мітозу Telophase of mitosis

Постмітотичний період інтерфази Postmitotic interphase period

Анафаза мітозу Mitosis Anaphase

Синтетичний період Synthetic period

126 / 200
У чоловіка 35-ти років під час тривалого бігу виникла гостра серцева недостатність. Які зміни іонного складу спостерігаються у серцевому м’язі при цьому стані? A 35-year-old man developed acute heart failure during a long run. What changes in the ion composition are observed in the heart muscle during this condition?

Зменшення в позаклітинному просторі іонів K+ і Mg2+ Decrease in the extracellular space of K+ and Mg2+ ions

Накопичення в клітинах міокарда іонів Na+ і Ca2+ Accumulation of Na+ and Ca2+ ions in myocardial cells

Накопичення в клітинах міокарда іонів K+ і Mg2+ Accumulation of K+ and Mg2+ ions in myocardial cells

Збільшення в позаклітинному просторі іонів Na+ і Ca2+ Increase in the extracellular space of Na+ and Ca2+ ions

Зменшення в клітинах міокарда іонів Na+ і Ca2+ Reduction of Na+ and Ca2+ ions in myocardial cells

127 / 200
Під час аналізу ЕКГ людини з’ясовано, що у другому стандартному відведенні від кінцівок зубці P позитивні, їхня амплітуда 0,1 mV (норма - 0,05-0,25 mV), тривалість - 0,1 с (норма - 0,07-0,10 с). Вірним є висновок, що у передсердях нормально відбувається процес: During the analysis of the human ECG, it was found that in the second standard lead from the limbs, P waves are positive, their amplitude is 0.1 mV (norm - 0.05- 0.25 mV), duration - 0.1 s (norm - 0.07-0.10 s). The correct conclusion is that the process occurs normally in the atria:

Реполяризації Repolarizations

Скорочення Abbreviation

Збудження Excitement

Деполяризації Depolarizations

Розслаблення Relax

128 / 200
У юнака 18-ти років діагностовано хворобу Марфана. При дослідженні встановлено: порушення розвитку сполучної тканини, будови кришталика ока, аномалії серцевосудинної системи, арахнодактилія. Яке генетичне явище зумовило розвиток цієї хвороби? An 18-year-old young man was diagnosed with Marfan's disease. During the examination, it was found: a violation of the development of connective tissue, the structure of the lens of the eye, anomalies of the cardiovascular system, arachnodactyly. What genetic phenomenon caused the development of this disease?

Неповне домінування Incomplete dominance

Комплементарність Complementarity

Плейотропія Pleiotropy

Кодомінування Codominance

Множинний алелізм Multiple allelism

129 / 200
Інозитолтрифосфати в тканинах організму утворюються в результаті гідролізу фосфатидилінозитолдифосфатів і відіграють роль вторинних посередників (месенджерів) в механізмі дії гормонів. Їхній вплив у клітині спрямований на: Inositol triphosphates in body tissues are formed as a result of hydrolysis of phosphatidylinositol diphosphates and play the role of secondary mediators (messengers) in the mechanism of hormone action. Their influence in the cell is aimed at:

Активацію протеїнкінази А Activation of protein kinase A

Активацію аденілатциклази Activation of adenylate cyclase

Гальмування протеїнкінази С Inhibition of protein kinase C

Гальмування фосфодіестерази Inhibition of phosphodiesterase

Вивільнення іонів кальцію з клітинних депо Release of calcium ions from cellular depots

130 / 200
Батьки дитини 3-х років звернули увагу на потемніння кольору його сечі при відстоюванні. Об’єктивно: температура у нормі, шкірні покриви чисті, рожеві, печінка не збільшена. Назвіть імовірну причину даного стану: The parents of a 3-year-old child drew attention to the darkening of the color of his urine during defecation. Objectively: the temperature is normal, the skin is clean, pink, the liver is not enlarged Name the probable cause of this condition:

Подагра Gout

Синдром Іценка-Кушінга Itsenko-Cushing syndrome

Гемоліз Hemolysis

Фенілкетонурія Phenylketonuria

Алкаптонурія Alkaptonuria

131 / 200
Хворий звернувся до лікаря зі скаргами на гнійничкові висипання на шкірі кінцівок. Який антисептик необхідно призначити хворому? The patient turned to the doctor with complaints of pustular rashes on the skin of the limbs. What antiseptic should be prescribed to the patient?

Гепарин Heparin

Преднізолон Prednisone

Сибазон Sibazon

Інсулін Insulin

Розчин йоду спиртовий Iodine alcohol solution

132 / 200
Після тривалого фізичного навантаження під час заняття з фізичної культури у студентів розвинулась м’язова крепатура. Причиною її виникнення стало накопичення у скелетних м’язах молочної кислоти. Вона утворилась після активації в організмі студентів: After prolonged physical exertion during a physical education class, the students developed muscle stiffness. The cause of its occurrence was the accumulation of lactic acid in the skeletal muscles. It was formed after activation in the body of students:

Гліколізу Glycolysis

Ліполізу Lipolysis

Глюконеогенезу Gluconeogenesis

Пентозофосфатного циклу Pentose phosphate cycle

Глікогенезу Glycogenesis

133 / 200
Під дією негативних чинників довкілля порушена функція міосателітоцитів. Зміну якої функції всього м’язового волокна слід очікувати в даному випадку? Under the influence of negative environmental factors, the function of myosatellitocytes is disturbed. What change in the function of the entire muscle fiber should be expected in this case?

Розслаблення Relax

Скоротливий термогенез Short Thermogenesis

Регенерація Regeneration

Скорочення Abbreviation

Трофіка Trofics

134 / 200
У хворого сечокам’яна хвороба. При видаленні конкременту з правого сечоводу хірург розрізав стінку сечоводу. В яке анатомічне утворення потрапить сеча? The patient has urolithiasis. When removing a calculus from the right ureter, the surgeon cut the wall of the ureter. What anatomical formation will the urine enter?

Заочеревинний простір Retroperitoneal space

Правий брижовий синус Right mesenteric sinus

Лівий бічний канал Left Side Channel

Прямокишково-міхурове заглиблення Rectal-bladder recess

Правий бічний канал Right Side Channel

135 / 200
У дитячому колективі проведено планову вакцинацію проти кору. Яким методом можна перевірити ефективність проведеної вакцинації? A scheduled vaccination against measles was carried out in the children's team. What method can be used to check the effectiveness of the vaccination?

Алергопроба Allergy test

Вірусоскопічний Virusoscopic

Вірусологічний Virus

Біологічний Biological

Серологічний Serological

136 / 200
У тварини збільшений тонус м’язів-розгиначів. Це є наслідком посиленої передачі інформації до мотонейронів спинного мозку такими низхідними шляхами: The animal has an increased tone of the extensor muscles. This is a consequence of the increased transmission of information to the motoneurons of the spinal cord by the following descending pathways:

Медіальні кортикоспінальні Medial corticospinal

Ретикулоспінальні Reticulospinal

Латеральні кортикоспінальні Lateral corticospinal

Руброспінальні Rubrospinal

Вестибулоспінальні Vestibulospinal

137 / 200
Хвора 28-ми років потрапила до інфекційної лікарні з приводу пожовтіння шкіри, склер, слизових оболонок. Лабораторно встановлене підвищення рівня прямого білірубіну у крові. В сечі виявлений уробіліноген і білірубін. Для якого з перелічених захворювань характерні такі зміни? A 28-year-old patient was admitted to an infectious disease hospital due to yellowing of the skin, sclera, and mucous membranes. An increase in the level of direct bilirubin in the blood was found in the laboratory. Urobilinogen and bilirubin. Which of the listed diseases is characterized by such changes?

Туберкульоз нирки Kidney tuberculosis

Інфаркт нирки Kidney infarction

Паренхіматозна жовтяниця Parenchymatous jaundice

Гемолітична жовтяниця Hemolytic Jaundice

Механічна жовтяниця Mechanical jaundice

138 / 200
У новонародженої дитини спостерігаються: судоми, блювання, жовтяниця, специфічний запах сечі. Лікар-генетик висловив підозру про спадкову хворобу обміну речовин. Який метод дослідження необхідно використати для постановки точного діагнозу? A newborn child has: convulsions, vomiting, jaundice, a specific smell of urine. A geneticist has suspected a hereditary metabolic disease. What research method should be used to establish exact diagnosis?

Біохімічний Biochemical

Популяційно-статистичний Population-statistical

Дерматогліфіка Dermatoglyphics

Близнюковий Twin

Цитогенетичний Cytogenetic

139 / 200
У дитини спостерігається затримка фізичного та розумового розвитку, глибокі порушення з боку сполучної тканини внутрішніх органів; у сечі виявлено кератансульфати. Обмін яких речовин порушений? The child has a delay in physical and mental development, deep disorders of the connective tissue of internal organs; keratan sulfates were detected in the urine. The exchange of what substances is disturbed?

Фібронектин Fibronectin

Еластин Elastin

Колаген Collagen

Глікозаміноглікани Glycosaminoglycans

Гіалуронова кислота Hyaluronic acid

140 / 200
У деяких анаеробних бактерій піруват, що утворюється внаслідок гліколізу, перетворюється на етиловий спирт (спиртове бродіння). У чому біологічний сенс цього процесу? In some anaerobic bacteria, pyruvate, formed as a result of glycolysis, turns into ethyl alcohol (alcoholic fermentation). What is the biological meaning of this process?

Поповнення фонду НАД+ Replenishment of the NAD+ fund

Забезпечення клітини НАДФН Provision of NADPH cell

Утворення лактату Lactate Formation

Утворення АДФ Formation of ADP

Утворення АТФ ATP formation

141 / 200
У хворого, який довготривало приймав преднізолон, в результаті відміни препарату виникло загострення захворювання, зниження артеріального тиску, слабкість. З чим можна пов’язати ці прояви? A patient who has been taking prednisolone for a long time, as a result of withdrawal of the drug, there was an exacerbation of the disease, a decrease in blood pressure, weakness. What can be associated with these manifestations?

Гіперпродукція АКТГ Hyperproduction of ACTH

Звикання до препарату Addiction to the drug

Виникнення недостатності кори наднирників Emergence of adrenal cortex insufficiency

Кумуляція препарату Cumulation of drug

Сенсибілізація до препарату Sensitization to the drug

142 / 200
В експерименті необхідно оцінити рівень збудливості тканини. Для цього доцільно визначити: In the experiment, it is necessary to estimate the level of tissue excitability. For this, it is advisable to determine:

Амплітуду ПД Amplitude of PD

Поріг деполяризації Depolarization Threshold

Критичний рівень деполяризації Critical level of depolarization

Тривалість ПД Duration of PD

Потенціал спокою Rest potential

143 / 200
Працівник соціальної служби після тривалого спілкування з людиною без визначеного місця проживання захворів на туберкульоз. До складу комплексного лікування було включено напівсинтетичний антибіотик широкого спектру дії. Вкажіть препарат: A social service worker fell ill with tuberculosis after long-term communication with a person without a fixed place of residence. A semi-synthetic broad-spectrum antibiotic was included in the complex treatment. Specify the drug:

Лінкоміцин Lincomycin

Ампіцилін Ampicillin

Еритроміцин Erythromycin

Рифампіцин Rifampicin

Цефотаксим Cefotaxime

144 / 200
При хворобі Вільсона-Коновалова порушується транспорт міді, що призводить до накопичення цього металу в клітинах мозку та печінки. З порушенням синтезу якого білку це пов’язано? In Wilson-Konovalov disease, the transport of copper is disturbed, which leads to the accumulation of this metal in the cells of the brain and liver. This is associated with a violation of the synthesis of which protein?

Транскобаламін Transcobalamin

Церулоплазмін Ceruloplasmin

Металотіонеїн Metallothionein

Сидерофілін Siderophilin

Гаптоглобін Haptoglobin

145 / 200
Хворий впродовж трьох років безрезультатно лікувався з приводу значного зниження кислотності шлункового соку. Його пригнічувала поява на білизні, постелі члеників, що рухались і самостійно виповзали з анального отвору. Який найбільш імовірний діагноз? For three years, the patient was unsuccessfully treated for a significant decrease in the acidity of gastric juice. He was oppressed by the appearance on his underwear and bed of members that moved and independently crawled out of the anus. What the most likely diagnosis?

Цистицеркоз Cysticercosis

Гіменолепідоз Hymenolepidosis

Опісторхоз Opistorchosis

Теніаринхоз Taeniarhynchosis

Теніоз Taeniosis

146 / 200
При нестачі біотину спостерігається порушення синтезу вищих жирних кислот. Утворення якого із зазначених метаболітів може бути порушено при цьому? With a lack of biotin, a violation of the synthesis of higher fatty acids is observed. The formation of which of the indicated metabolites can be disturbed in this case?

Аланін Аланін

Малоніл КоА Malonil CoA

Серотонін Serotonin

Сукциніл КоА Succinyl CoA

Піруват Pyruvate

147 / 200
У чоловіка 60-ти років після інсульту настав тривалий сон. Ураження яких структур ЦНС найбільш імовірно призвело до цього стану? A 60-year-old man had a long sleep after a stroke. Damage to which CNS structures most likely led to this condition?

Мозочок Cerebellum

Чорна субстанція Black substance

Висхідна частина РФ Eastern part of the Russian Federation

V-IX пари черепних нервів V-IX pairs of cranial nerves

Прецентральна звивина Precentral gyrus

148 / 200
У чоловіка виявлене захворювання, яке зумовлене домінантним геном, локалізованим у Ххромосомі. У кого із дітей буде це захворювання, якщо дружина здорова? The husband has been diagnosed with a disease caused by a dominant gene localized in the Chromosome. Which of the children will have this disease if the wife is healthy?

У всіх дітей All children have

Тільки у синів Only sons

У половини дочок Half daughters

У половини синів Half sons

Тільки у дочок Only in daughters

149 / 200
Недостатність в організмі мікроелементу селену проявляється кардіоміопатією. Імовірною причиною такого стану є зниження активності такого селенвмісного ферменту: Deficiency of the trace element selenium in the body is manifested by cardiomyopathy. The probable cause of this condition is a decrease in the activity of this selenium-containing enzyme:

Глутатіонпероксидаза Glutathione peroxidase

Сукцинатдегідрогеназа Succinate dehydrogenase

Каталаза Catalase

Лактатдегідрогеназа Lactate dehydrogenase

Цитохромоксидаза Cytochrome oxidase

150 / 200
Недостатність в організмі лінолевої та ліноленової кислот призводить до ушкоджень шкіри, випадіння волосся, сповільненого загоювання ран, тромбоцитопенії, зниження опірності до інфекційних захворювань. Порушення синтезу яких речовин найімовірніше зумовлює вказані симптоми? Insufficiency of linoleic and linolenic acids in the body leads to skin damage, hair loss, delayed wound healing, thrombocytopenia, reduced resistance to infectious diseases. Violation of the synthesis of which substances most likely causes indicated symptoms?

Інтерлейкіни Interleukins

Кортикостероїди Corticosteroids

Катехоламіни Catecholamines

Ейкозаноїди Eicosanoids

Інтерферони Interferons

151 / 200
Людина потрапила у крижану воду й швидко загинула в результаті різкого переохолодження. Це відбулося тому, що в даному випадку значно збільшилась віддача тепла організмом таким шляхом: A person fell into icy water and quickly died as a result of severe hypothermia. This happened because in this case the body's heat transfer increased significantly in the following way:

Конвекція Convection

- -

Теплопроведення і радіація Heat conduction and radiation

Радіація Radiation

Теплопроведення Heat conduction

152 / 200
Хворий 39-ти років з алкогольним цирозом печінки скаржиться на задишку, загальну слабкість. Встановлено зниження артеріального тиску, розширення поверхневих вен передньої стінки живота, спленомегалію. Яке порушення гемодинаміки спостерігається у хворого? A 39-year-old patient with alcoholic cirrhosis of the liver complains of shortness of breath, general weakness. Decreased blood pressure, dilation of the superficial veins of the anterior abdominal wall, splenomegaly were found. What a hemodynamic disorder observed in the patient?

Тотальна серцева недостатність Total heart failure

Недостатність правого шлуночка серця Right ventricular failure

Колапс Collapse

Синдром портальної гіпертензії Portal hypertension syndrome

Недостатність лівого шлуночка серця Left ventricular failure

153 / 200
У жінки, що тривало приймала антибіотики з приводу кишкової інфекції, розвинулось ускладнення з боку слизової порожнини рота у вигляді запального процесу і білого нальоту, у якому під час бактеріологічного дослідження були виявлені дріжджеподібні грибки Candida albicans. Який з перерахованих препаратів показаний для лікування цього ускладнення? A woman who had been taking antibiotics for an intestinal infection for a long time developed a complication from the mucous membrane of the mouth in the form of an inflammatory process and a white plaque, in which during a bacteriological examination yeast-like fungi Candida albicans were detected. Which of the listed drugs is indicated for the treatment of this complication?

Фуразолідон Furazolidone

Тетрациклін Tetracycline

Флуконазол Fluconazole

Бісептол Biseptol

Поліміксин Polymixin

154 / 200
До лікаря акушера-гінеколога звернулась вагітна жінка, у якої діагностували мегалобластну анемію. Який з нижченаведених засобів доцільно призначити? A pregnant woman who was diagnosed with megaloblastic anemia consulted an obstetrician-gynecologist. Which of the following drugs should be prescribed?

Пентоксил Pentoxyl

Ілауцин Ілауцин

Стрептокіназа Streptokinase

Ціанокобаламін Cyanocobalamin

Метилурацил Methyluracil

155 / 200
Хвора 45-ти років звернулась із скаргами на облисіння. При огляді: шкіра голови плямисто-коричневого кольору, малорухлива, тоненька, щільна, вкрита роговими лусочками. При гістологічному дослідженні - гіперкератоз, в дермі склероз, периваскулярні лімфомакрофагальні інфільтрати, атрофія потових та сальних залоз. У крові виявлені LE-клітини. Який найбільш імовірний діагноз? A 45-year-old patient complained of baldness. On examination: the scalp is mottled-brown in color, inactive, thin, dense, covered with horny scales. At histological examination in the examination - hyperkeratosis, sclerosis in the dermis, perivascular lympho-macrophagic infiltrates, atrophy of sweat and sebaceous glands. LE cells were detected in the blood. What is the most likely diagnosis?

Ревматизм Rheumatism

Системний червоний вовчак Systemic lupus erythematosus

Ревматоїдний артрит Rheumatoid arthritis

Системна склеродермія Systemic Scleroderma

Вузликовий періартеріїт Nodular periarteritis

156 / 200
При дослідженні гостроти слуху в коваля виявили втрату слуху на 50% у діапазоні низьких частот і майже нормальну гостроту слуху в діапазоні високих частот. Порушення яких структур слухової системи призвело до такого стану? When examining the hearing acuity of a blacksmith, they found a 50% hearing loss in the low frequency range and almost normal hearing acuity in the high frequency range. Violation of which structures of the auditory system led to such a state?

Середня частина кортієвого органу Middle part of the organ of Corti

Барабанна перетинка Eardrum

Кортієв орган - ближче до гелікотреми Organ of Corti - closer to helicotrema

Кортієв орган - ближче до овального віконця Organ of Corti - closer to the oval window

М’язи середнього вуха Middle ear muscles

157 / 200
При відборі для ревакцинації вакциною БЦЖ у школяра поставлено пробу Манту, яка виявилася негативною. Результат проби свідчить про такі особливості імунітету до туберкульозу: During selection for revaccination with the BCG vaccine, a Mantoux test was performed on a schoolboy, which turned out to be negative. The test result indicates the following features of immunity to tuberculosis:

Відсутність антитоксичного імунітету Lack of antitoxic immunity

Відсутність гуморального імунітету Lack of humoral immunity

Відсутність клітинного імунітету Absence of cellular immunity

Наявність клітинного імунітету Availability of cellular immunity

Наявність гуморального імунітету Availability of humoral immunity

158 / 200
Чоловік звернувся до лікаря з приводу безпліддя. Має високий зріст, зниження інтелекту, недорозвинення статевих залоз. У епітелії слизової оболонки порожнини рота виявлений статевий хроматин (1 тільце Барра). Про яку патологію можна думати? A man consulted a doctor about infertility. He is tall, has reduced intelligence, underdeveloped gonads. In the epithelium of the mucous membrane of the oral cavity, sex chromatin was detected (1 Barr body) What pathology can you think about?

Акромегалія Acromegaly

Синдром Ді-Джорджи DiGiorgia Syndrome

Синдром Клайнфельтера Klinefelter Syndrome

Синдром Іценка-Кушинга Itsenko-Cushing syndrome

Адреногенітальний синдром Adrenogenital syndrome

159 / 200
Який стан може розвинутися через 15-30 хвилин після повторного введення антигену внаслідок підвищеного рівня антитіл, переважно IgE, які адсорбуються на поверхні клітин-мішеней - тканинних базофілів (тучних клітин) та базофілів крові? What condition can develop 15-30 minutes after re-introduction of an antigen due to an increased level of antibodies, mainly IgE, which are adsorbed on the surface of target cells - tissue basophils (fat cells) and blood basophils?

Анафілаксія Anaphylaxis

Сироваткова хвороба Serum sickness

Гіперчутливість уповільненого типу Delayed hypersensitivity

Імунно-комплексна гіперчутливість Immune complex hypersensitivity

Антитіло-залежна цитотоксичність Antibody-dependent cytotoxicity

160 / 200
У жінки 22-х років через 5 годин після вживання морепродуктів на шкірі тулуба та дистальних відділів кінцівок з’явились маленькі сверблячі папули, які частиною зливаються між собою. Через добу висипка самовільно зникла. Назвіть механізм гіперчутливості, що полягає в основі даних змін: In a 22-year-old woman, 5 hours after eating seafood, small itchy papules appeared on the skin of the trunk and distal parts of the limbs, which partially merge with each other. day the rash spontaneously disappeared. Name the mechanism of hypersensitivity underlying these changes:

Антитілоопосередкований клітинний цитоліз Antibody-mediated cellular cytolysis

Системна анафілаксія Systemic anaphylaxis

Клітинна цитотоксичність Cellular cytotoxicity

Атопія (місцева анафілаксія) Atopy (local anaphylaxis)

Імунокомплексна гіперчутливість Immune complex hypersensitivity

161 / 200
Хворий на гіпертонічну хворобу разом з безсольовою дієтою та з антигіпертензивними засобами, довгий час приймав гідрохлортіазид, що зумовило порушення електролітного балансу. Яке порушення внутрішнього середовища виникло у хворого? A hypertensive patient, along with a salt-free diet and antihypertensive drugs, has been taking hydrochlorothiazide for a long time, which caused a violation of the electrolyte balance. What violation of the patient's internal environment occurred?

Метаболічний ацидоз Metabolic acidosis

Гiпермагнiємiя Hypermagnesemia

Гіпохлоремічний алкалоз Hypochloremic alkalosis

Збільшення об’єму циркулюючої крові Increasing the volume of circulating blood

Гiперкалiємiя Hyperkalemia

162 / 200
У 49-річної жінки після видалення лімфатичних вузлів правої аксилярної ділянки з приводу раку грудної залози через півроку виявлено збільшення правої верхньої кінцівки в об’ємі та значне її ущільнення, гладка та напружена шкіра, через щілиноподібні дефекти якої на поверхню витікає прозора рідина. Діагностуйте вид порушення лімфообігу: In a 49-year-old woman, after the removal of the lymph nodes of the right axillary region due to breast cancer, six months later, an increase in the volume of the right upper limb and its significant compaction were found, smooth and tense skin, through the slit-like defects of which a clear liquid flows to the surface. Diagnose the type of lymphatic circulation disorder:

Хронічна набута місцева лімфедема Chronic acquired local lymphedema

Гостра місцева лімфедема Acute local lymphedema

Гостра загальна лімфедема Acute generalized lymphedema

Хронічна загальна лімфедема Chronic general lymphedema

Хронічна вроджена місцева лімфедема Chronic congenital local lymphedema

163 / 200
У сироватці хворого виявлений імуноферментним методом HBsAg. При якому захворюванні виявлення даного ангигену має діагностичне значення? HBsAg was detected in the patient's serum by the immunoenzymatic method. In which disease does the detection of this antigen have diagnostic value?

ВІЛ ВІЛ

Сказ Tale

Натуральна віспа Smallpox

Кір Кір

Вірусний гепатит В Viral hepatitis B

164 / 200
При повторному введенні алергену починається виділення гістаміну тучними клітинами крові. До якого рівня реактивності відноситься така відповідь організму? When the allergen is re-introduced, the release of histamine by blood mast cells begins. To what level of reactivity does this response of the body belong?

Субклітинний Subcellular

Молекулярний Molecular

Системний System

Органний Organ

Клітинний Cellular

165 / 200
В експерименті на кролику введення пірогеналу призвело до підвищення у тварини температури тіла. Яка з перерахованих речовин відіграє роль вторинного пірогену, що бере участь у механізмі виникнення лихоманкової реакції? In an experiment on a rabbit, the introduction of pyrogenal led to an increase in the animal's body temperature. Which of the listed substances plays the role of a secondary pyrogen involved in the mechanism of febrile reaction?

Інтерлейкін-1 Interleukin-1

Брадикінін Brady's

Піромен Piromen

Імуноглобулін Immunoglobulin

Гістамін Histamine

166 / 200
При авторадіографічному дослідженні епітелію тонкої кишки було виявлено, що його повне оновлення відбувається протягом 3-х діб за рахунок проліферації малодиференційованих клітин. Вкажіть їх локалізацію: During the autoradiographic study of the epithelium of the small intestine, it was found that its complete renewal occurs within 3 days due to the proliferation of poorly differentiated cells. Specify their localization:

Дно крипт Crypt Bottom

Бічна поверхня ворсинок Lateral surface of villi

Верхівка ворсинок Top of villi

Основа ворсинок Base of villi

Власна пластинка слизової оболонки Own plate of the mucous membrane

167 / 200
Лікар при дослідженні мазку крові у пацієнта з анемією встановив діагноз - спадкова гемолітична анемія Мінковського-Шофара. Виявлення у крові яких характерних клітин надало можливість лікарю встановити діагноз? When examining a blood smear in a patient with anemia, the doctor made a diagnosis of hereditary hemolytic anemia of Minkovsky-Shofar. The detection of which characteristic cells in the blood made it possible for the doctor to make the diagnosis?

Мегалоцити Megalocytes

Мікросфероцити Microspherocytes

Поліхроматофіли Polychromatophiles

Пойкілоцити Poikilocytes

Анізоцити Anisocytes

168 / 200
У хворого з клінічними симптомами гіпотиреозу, щитоподібна залоза збільшена удвічі, при пальпації щільна, з горбистою поверхнею. При гістологічному дослідженні - поряд з атрофією фолікулів залози відмічається дифузна інфільтрація паренхіми лімфоцитами, плазматичними клітинами з утворенням фолікулів і посилене розростання сполучної тканини. Вкажіть найбільш імовірний діагноз: In a patient with clinical symptoms of hypothyroidism, the thyroid gland is doubled in size, dense on palpation, with a bumpy surface. During histological examination, along with atrophy of the gland follicles, diffuse infiltration of the parenchyma is noted lymphocytes, plasma cells with the formation of follicles and increased growth of connective tissue. Specify the most likely diagnosis:

Спорадичний зоб Sporadic goiter

Фіброзний зоб Fibrous goiter

Зоб Хашімото Hashimoto's goiter

Ендемічний зоб Endemic goiter

Дифузний токсичний зоб Diffuse toxic goiter

169 / 200
При загостренні ревматоїдного артриту хворому, в анамнезі якого супутній хронічний гастрит, призначений целекоксиб. Чим обумовлено зменшення побічної дії препарату на травний тракт? For an exacerbation of rheumatoid arthritis, a patient with a history of accompanying chronic gastritis is prescribed celecoxib. What is the reason for the reduction of side effects of the drug on the digestive tract?

Переважаюча стимуляція аденіла-тциклази Predominant stimulation of adenyl-tcyclase

Переважаюче пригнічення циклооксигенази-1 Predominant inhibition of cyclooxygenase-1

Пригнічення фосфоліпази А2 Inhibition of phospholipase A2

Пригнічення фосфодіестерази Inhibition of phosphodiesterase

Переважаюче пригнічення циклооксигенази-2 Predominant inhibition of cyclooxygenase-2

170 / 200
В експерименті на кролі встановлено, що об’єм кисню, який споживається головним мозком за 1 хвилину, дорівнює об’єму CO2 , який виділяється клітинами мозку в кров. Це свідчить, що у клітинах головного мозку має місце: In an experiment on a rabbit, it was established that the volume of oxygen consumed by the brain in 1 minute is equal to the volume of CO2 released by brain cells into the blood. This shows that in the cells of the brain there is:

Гіпокапнія Hypocapnia

Окиснення білків Protein oxidation

Окиснення вуглеводів Oxidation of carbohydrates

Гіпоксія Hypoxia

Окиснення жирів Fat oxidation

171 / 200
При активації запального процесу, деяких аутоімунних та інфекційних захворюваннях у плазмі крові різко зростає рівень білків гострої фази. Який із наведених нижче білків здатний утворювати гель при охолодженні сироватки? During the activation of the inflammatory process, some autoimmune and infectious diseases, the level of acute phase proteins in the blood plasma increases sharply. Which of the following proteins is able to form a gel when the serum is cooled?

Церулоплазмін Ceruloplasmin

Кріоглобулін Cryoglobulin

Гаптоглобін Haptoglobin

а2-макроглобін a2-macroglobin

С-реактивний білок C-reactive protein

172 / 200
Хворому поставлено діагноз газова гангрена. Після ідентифікації збудника досліджуваний матеріал необхідно знищити. Який метод слід використати? The patient was diagnosed with gas gangrene. After identification of the causative agent, the research material must be destroyed. What method should be used?

Тиндалізація Tyndalization

Кип’ятіння Boiling

Стерилізація текучою парою Steam sterilization

Пастеризація Pasteurization

Стерилізація парою під тиском Steam sterilization under pressure

173 / 200
Фенілкетонурія - це захворювання, яке зумовлено рецесивним геном, що локалізується в аутосомі. Батьки є гетерозиготами за цим геном. Вони вже мають двох хворих синів і одну здорову доньку. Яка імовірність, що четверта дитина, яку вони очікують, народиться теж хворою? Phenylketonuria is a disease caused by an autosomal recessive gene. The parents are heterozygous for this gene. They already have two sick sons and one healthy daughter. What is the probability that the fourth child they are expecting will also be born sick?

0% 0%

100% 100%

50% 50%

75% 75%

25% 25%

174 / 200
У пацієнта з підвищеним артеріальним тиском, тремором, тахікардією, була діагностовано доброякісна пухлина мозкової речовини наднирників. Гіперсекреція якого гормону викликає таку симптоматику? A benign tumor of the adrenal medulla was diagnosed in a patient with high blood pressure, tremors, and tachycardia. Hypersecretion of which hormone causes such symptoms?

Адреналін Adrenaline

Глюкагон Glucagon

Інсулін Insulin

Тироксин Thyroxine

Соматотропін Somatotropin

175 / 200
В пробірку, що містить розчин NaCl 0,9%, додали краплю крові. Що відбудеться з еритроцитами? A drop of blood was added to a test tube containing a 0.9% NaCl solution. What will happen to the erythrocytes?

Зморшкування Wrinkle

Осмотичний гемоліз Osmotic hemolysis

Набухання Swelling

Залишаться без змін Remain unchanged

Біологічний гемоліз Biological hemolysis

176 / 200
До косметолога звернулася пацієнтка зі скаргами на появу чорних цяток на обличчі. Після обстеження було встановлено, що поява цяток пов’язана з порушенням виділення секрету сальних залоз. Який тип секреції характерний для цих залоз? A patient turned to a cosmetologist with complaints about the appearance of black spots on her face. After the examination, it was established that the appearance of spots is associated with a violation of the secretion of the sebaceous glands. What type secretion characteristic of these glands?

Мікроапокриновий Microapocrine

Мерокриновий та мікроапокриновий Merocrine and microapocrine

Мерокриновий Merocrine

Голокриновий Holocrine

Макроапокриновий Macroapocrynovial

177 / 200
У пацієнта з хронічним захворюванням нирок розвинулась ниркова недостатність. Який з показників найбільш імовірно свідчить про порушення реабсорбції в канальцях в даному випадку? A patient with chronic kidney disease developed kidney failure. Which of the indicators most likely indicates impaired reabsorption in the tubules in this case?

Гіпо- та ізостенурія Hyposthenuria

Зниження кліренсу Decreasing clearance

Лейкоцитурія Leukocyturia

Гематурія Hematuria

Гіперазотемія Hyperazotemia

178 / 200
У людей, які постійно проживають в гірській місцевості, адаптація до 'кисневого голодування' здійсню ється шляхом полегшеної віддачі кисню гемоглобіном внаслідок: In people who live permanently in mountainous areas, adaptation to 'oxygen starvation' is carried out by the facilitated return of oxygen by hemoglobin as a result of:

Зниження температури крові Decreasing blood temperature

Зростання парціального тиску CO2 Increasing CO2 partial pressure

Зниженого утворення 2,3- дифосфогліцерату в еритроцитах Decreased formation of 2,3-diphosphoglycerate in erythrocytes

Підвищення pH крові Blood pH increase

Підвищеного утворення 2,3- дифосфогліцерату в еритроцитах Increased formation of 2,3-diphosphoglycerate in erythrocytes

179 / 200
У хворого 40-ка років ознаки гірської хвороби: запаморочення, задишка, тахікардія, рН крові - 7,50, pCO2 -30 мм рт.ст., зсув буферних основ +4 ммоль/л. Яке порушення кислотноосновного стану має місце? A 40-year-old patient has signs of mountain sickness: dizziness, shortness of breath, tachycardia, blood pH - 7.50, pCO2 -30 mm Hg, shift buffer bases +4 mmol/l. What violation of the acid-base state is taking place?

Газовий алкалоз Gas alkalosis

Негазовий алкалоз Nongaseous alkalosis

Газовий ацидоз Gas acidosis

Видільний ацидоз Excretory acidosis

Негазовий ацидоз Nongaseous acidosis

180 / 200
Після ремонту автомобіля в закритому приміщенні при працюючому двигуні у чоловіка з’явилися задишка, запаморочення, акроціаноз, частота дихання 24-26/хв. Газовий склад крові: pO2 - 60 мм рт.ст., pCO2 - 30 мм рт.ст.; у крові наявний карбоксигемоглобін. Про який вид гіпоксії можна думати? After repairing the car in a closed room with the engine running, the man developed shortness of breath, dizziness, acrocyanosis, respiratory rate 24-26/min. Blood gas composition: pO2 - 60 mm Hg, pCO2 - 30 mm Hg; carboxyhemoglobin is present in the blood. What kind of hypoxia can you think about?

Гемічна Гемічна

Гіпоксична Hypoxic

Тканинна Fabric

Циркуляторна Circular

Респіраторна Respiratory

181 / 200
При обстеженні у хворого виявлене порушення чутливості шкіри в ділянці передньої поверхні шиї. Який нерв уражений? During the examination, the patient was found to have a skin sensitivity disorder in the area of the front surface of the neck. Which nerve is affected?

Надключичні Supraclavicular

Поперечний нерв шиї Transverse neck nerve

Шийна петля Sheena's loop

Великий вушний Big ear

Малий потиличний Small occipital

182 / 200
У вагітної жінки 26-ти років після тривалого блювання було зареєстровано зниження об’єму циркулюючої крові. Про яку зміну загальної кількості крові може йти мова? A 26-year-old pregnant woman had a decrease in the volume of circulating blood after prolonged vomiting. What kind of change in the total amount of blood can we be talking about?

Проста гіповолемія Simple hypovolemia

Олігоцитемічна гіповолемія Oligocythemic hypovolemia

Поліцитемічна гіперволемія Polycythemic hypervolemia

Поліцитемічна гіповолемія Polycythemic hypovolemia

Олігоцитемічна гіперволемія Oligocythemic hypervolemia

183 / 200
У хворої дитини гінгівіт, спричинений анаеробною інфекцією. Яку групу протимікробних засобів потрібно призначити для лікування? A sick child has gingivitis caused by an anaerobic infection. What group of antimicrobial agents should be prescribed for treatment?

Сульфаніламіди Sulfonamides

Нітроімідазоли Nitroimidazoles

Аміноглікозиди Aminoglycosides

Поліміксини Polymyxins

Нітрофурани Nitrofurans

184 / 200
Пацієнту, який знаходився в клініці з приводу пневмонії, ускладненої плевритом, у складі комплексної терапії вводили преднізолон. Протизапальна дія цього синтетичного глюкокортикоїда пов’язана з блокуванням вивільнення арахідонової кислоти шляхом гальмування такого ферменту: A patient who was in the clinic for pneumonia complicated by pleurisy was administered prednisolone as part of complex therapy. The anti-inflammatory effect of this synthetic glucocorticoid is associated with blocking the release of arachidonic acid by inhibiting such an enzyme:

Ліпоксигеназа Lipoxygenase

Фосфоліпаза А2 Phospholipase A2

Пероксидаза Peroxidase

Циклооксигеназа Cyclooxygenase

Фосфоліпаза C Phospholipase C

185 / 200
Встановлено, що в клітинах організмів відсутні мембранні органели та їх спадковий матеріал не має нуклеосомної організації. Що це за організми? It was established that the cells of organisms lack membrane organelles and their hereditary material does not have a nucleosome organization. What kind of organisms are these?

Віруси Viruses

Аскоміцети Ascomicity

Прокаріоти Prokaryotes

Найпростіші The easiest

Еукаріоти Eukaryotes

186 / 200
У постраждалого в автомобільній аварії припинилося грудне дихання при збереженні діафрагмального. На якому рівні найбільш імовірно пошкоджено спинний мозок? A victim in a car accident stopped chest breathing while preserving the diaphragm. At what level is the spinal cord most likely damaged?

VI-VII шийні сегменти VI-VII cervical segments

XI-XII грудні сегменти XI-XII chest segments

I-II поперекові сегменти I-II lumbar segments

I-II шийні сегменти I-II cervical segments

I-II крижові сегменти I-II sacral segments

187 / 200
Хлопчик на другому році життя став часто хворіти на респіраторні захворювання, стоматити, гнійничкові ураження шкіри. Навіть невеликі пошкодження ясен і слизової ускладнюються запаленням, що протікає тривало. Встановлено, що у крові дитини практично відсутні імуноглобуліни усіх класів. Зниження функціональної активності якої клітинної популяції лежить в основі описаного синдрому? In the second year of his life, the boy often began to suffer from respiratory diseases, stomatitis, pustular skin lesions. Even minor damage to the gums and mucous membrane is complicated by inflammation that continues for a long time. It was established that that there are practically no immunoglobulins of all classes in the child's blood. A decrease in the functional activity of which cell population underlies the described syndrome?

Нейтрофіли Neutrophils

Макрофаги Macrophages

В-лімфоцити B-lymphocytes

Т-лімфоцити T-lymphocytes

NK-лімфоцити NK-lymphocytes

188 / 200
На електронній фотографії представлена органела, що являє собою великий поліпротеазний комплекс, що складається з трубкоподібної та двох регуляторних частин, які розташовані на обох кінцях органели. Остання виконує функцію протеолізу. Назвіть цю органелу: The electronic photo shows an organelle, which is a large polyprotease complex consisting of a tubular and two regulatory parts, which are located at both ends of the organelle. The latter performs the function of proteolysis . Name this organelle:

Рибосома Ribosome

Центріоль Centriol

Включення Inclusion

Протеасома Proteasome

Комплекс Гольджі Golgi Complex

189 / 200
У хворого після перенесеної черепно-мозкової травми порушений акт ковтання. Який відділ мозку постраждав? The patient has a disturbed act of swallowing after a brain injury. Which part of the brain is affected?

Кінцевий мозок End Brain

Середній мозок Average brain

Довгастий мозок Olongate brain

Таламус Thalamus

Проміжний мозок Midbrain

190 / 200
Дитина 6-ти років знаходиться на стаціонарному лікуванні з діагнозом алергічного риніту. В крові: зміни в лейкоцитарній формулі. Кількість яких клітин лейкоцитарного ряду може бути збільшена? A 6-year-old child is undergoing inpatient treatment with a diagnosis of allergic rhinitis. In the blood: changes in the leukocyte formula. The number of which cells of the leukocyte row can be increased?

Т-лімфоцити T-lymphocytes

Нейтрофіли Neutrophils

В-лімфоцити B-lymphocytes

Еозинофіли Eosinophils

Базофіли Basophils

191 / 200
У хворого 69-ти років на шкірі в ділянці нижньої повіки з’явилося невелике бляшкоподібне утворення з наступним виразкуванням, яке було оперативно видалене. При мікроскопічному дослідженні утворення: в дермі шкіри комплекси з атипових епітеліальних клітин; периферії клітини розташовані перпендикулярно до базальної мембрани. Клітини темні, призматичної полігональної форми з гіперхромними ядрами з частими мітозами. Іноді зустрічаються утворення, подібні до волосяного фолікула. Яка гістологічна форма рака у хворого? In a 69-year-old patient, a small plaque-like formation appeared on the skin in the area of the lower eyelid, followed by ulceration, which was surgically removed. Upon microscopic examination of the formation: in dermis of the skin, complexes of atypical epithelial cells; the periphery of the cells are located perpendicular to the basement membrane. Cells are dark, prismatic polygonal in shape with hyperchromic nuclei with frequent mitoses. Sometimes there are formations similar to a hair follicle. What is the histological form of cancer in the patient?

Аденокарцинома Adenocarcinoma

Недиференційований рак Undifferentiated cancer

Плоскоклітинний рак без ороговіння Squamous cell carcinoma without keratinization

Базально-клітинний рак Basal cell carcinoma

Плоскоклітинний рак з ороговінням Squamous cell carcinoma with keratinization

192 / 200
Анатомічний мертвий простір - це частина повітря, яка залишається в повітроносних шляхах після видиху. В якій із наведених нижче ситуацій відбудеться зменшення анатомічного мертвого простору? Anatomical dead space is the portion of air that remains in the airways after exhalation. In which of the following situations will the anatomical dead space decrease?

Дихання через рот Breathing through the mouth

Накладання трахеостоми Applying a tracheostomy

Поворот лежачого пацієнта на лівий бік Turn the lying patient to the left side

Поворот лежачого пацієнта на правий бік Turn the lying patient to the right side

Нахил голови вперед Head tilt forward

193 / 200
До кардіологічного відділення надійшов хворий з інтенсивним болем, який зумовлений інфарктом міокарда. Для купірування болю було вирішено потенціювати дію анальгетика нейролептиком. Який з перерахованих нейролептиків найбільш придатний у даному випадку? A patient came to the cardiology department with intense pain caused by a myocardial infarction. To relieve the pain, it was decided to potentiate the effect of the analgesic with a neuroleptic. Which of the listed neuroleptics is the most suitable in this case ?

Аміназин Aminazine

Сульпірид Sulpiride

Дроперидол Droperidol

Галоперидол Haloperidol

Трифтазин Triftazin

194 / 200
У 12-річної дитини непереносимість ряду харчових продуктів. Їх вживання викликає алергічну реакцію у вигляді висипань на шкірі, що сверблять. Який протигістамінний засіб слід призначити, щоб не заважати шкільним заняттям дитини? A 12-year-old child has an intolerance to a number of food products. Their use causes an allergic reaction in the form of itchy rashes on the skin. What antihistamine should be prescribed so as not to interfere the child's schoolwork?

Ефедрин Ephedrine

Еуфілін Euphilin

Димедрол Diphenhydramine

Лоратадин Loratadine

Диклофенак Diclofenac

195 / 200
У хворого на слизовій оболонці ясен виразка овальної форми з припіднятими краями хрящоподібної щільності. Дно виразки м’ясисто-червоного забарвлення з нашаруваннями сірого кольору. При мікроскопічному дослідженні - проліферація ендотелію дрібних судин, периваскулярна лімфоплазмоцитарна інфільтрація. Про яке захворювання йдеться? The patient has an oval-shaped ulcer on the mucous membrane of the gums with raised edges of cartilaginous density. The bottom of the ulcer is fleshy-red in color with gray layers. Microscopic examination shows proliferation endothelium of small vessels, perivascular lymphoplasmacytic infiltration. What disease are we talking about?

Травматична виразка Traumatic ulcer

Виразка-рак Virazka Cancer

Сифіліс Syphilis

Виразково-некротичний гінгівіт Ulcerous-necrotic gingivitis

Ерозивно-виразкова лейкоплакія Erosive-ulcerative leukoplakia

196 / 200
Стоматолог призначив пацієнту препарати кальцію для профілактики карієсу. Лікар не знав, що пацієнт хворіє на хронічну серцеву недостатність і приймає серцеві глікозиди. Що відбудеться в результаті поєднання препаратів кальцію і серцевих глікозидів? The dentist prescribed calcium preparations to the patient to prevent caries. The doctor did not know that the patient had chronic heart failure and was taking cardiac glycosides. What will happen as a result of the combination of calcium preparations and cardiac glycosides?

Знизиться активність і токсичність серцевих глікозидів The activity and toxicity of cardiac glycosides will decrease

Уповільниться метаболізм серцевих глікозидів The metabolism of cardiac glycosides will slow down

Збільшиться активність і токсичність серцевих глікозидів Activity and toxicity of cardiac glycosides will increase

Прискориться виведення серцевих глікозидів Excretion of cardiac glycosides will accelerate

Препарати не взаємодіють Drugs do not interact

197 / 200
Тривале лікування гіпофункції щитоподібної залози спричинило загальну дистрофію, карієс зубів, тахікардію, тремор кінцівок. Який лікарський засіб викликав зазначені побічні ефекти? Long-term treatment of hypothyroidism caused general dystrophy, dental caries, tachycardia, limb tremors. Which drug caused these side effects?

Паратиреоїдин Parathyroidin

Хумулін Humulin

Преднізолон Prednisone

L-тироксин L-thyroxine

Тирокальцитонін Thyrocalcitonin

198 / 200
У жінки встановлено діагноз - рак шийки матки. З яким вірусом може бути асоційована ця патологія? A woman has been diagnosed with cervical cancer. What virus can this pathology be associated with?

Аренавірус Arenavirus

Вірус простого герпеса тип 2 Herpes simplex virus type 2

Папілома вірус Papilloma virus

Цитомегаловірус Cytomegalovirus

Varicella-Zoster вірус Varicella-Zoster virus

199 / 200
У хворого хлопчика 12-ти років вміст холестерину в сироватці крові до 25 ммоль/л. В анамнезі - спадкова сімейна гіперхолестеринемія, причиною якої є порушення синтезу білків-рецепторів до: A sick 12-year-old boy has serum cholesterol up to 25 mmol/l. He has a history of hereditary familial hypercholesterolemia, the cause of which is a violation of the synthesis of receptor proteins to:

Ліпопротеїнів низької щільності Low-density lipoprotein

Ліпопротеїнів дуже низької щільності Very low density lipoprotein

Ліпопротеїнів високої щільності High-density lipoprotein

Ліпопротеїнів проміжної щільності Intermediate density lipoproteins

Хіломікронів Chylomicrons

200 / 200
При дослідженні зовнішнього дихання лікар попросив пацієнта здійснити максимально глибокий видих після максимально глибокого вдиху для визначення такого показника: During the study of external breathing, the doctor asked the patient to exhale as deeply as possible after inhaling as deeply as possible to determine the following indicator:

Життєва ємність легень Vital lung capacity

Резервний об’єм видиху Reserve expiratory volume

Функціональна залишкова ємність Functional residual capacity

Киснева ємність крові Blood oxygen capacity

Загальна ємність легень Total lung capacity